Main Index   Search   Register   Login   Who's Online   FAQ   Links
  1 Online, 0 Active   You are not logged in  
Main Index     The HIVE light edition (TM)
This is a historical archive
The forum is read-only. Private information has been removed. It is not possible to login.


Methods Discourse Thread:   Previous  Forum index  Next

All 91 posts   Subject: P2P compilation   Please login to post   Down

 
    Aurelius
(Hive Addict)
12-23-02 14:34
No 391997
      P2P compilation     

aurelius has decided to put forth a P2P and related compilation.  This time, unlike the aldehyde compilation, aurelius has decided that there will be much better organization even in the beginning- this way it can be much more easily used even at first.

suggestions are welcome. 
 
 
 
 
    Aurelius
(Hive Addict)
12-23-02 14:37
No 391998
      phenylacetonitriles to phenylacetones     

Rhodium
Phenylacetonitriles -> Phenylacetones
alpha-(3,4-Dimethoxyphenyl)-acetoacetonitrile

A solution of sodium ethylate was prepared from 389 g (17 moles) of sodium and 4530 cc. of absolute ethanol. To this hot solution was added a solution of 1000g (5.65 moles) of 3,4-dimethoxyphenylacetonitrile in 1710 g (19.4 moles) of dry ethyl acetate. After refluxing for 4 hours the mixture, containing a light precipitate, was allowed to stand at room temperature overnight. The white crystalline mass was filtered off after cooling in ice for 2 hours. Washing with ethyl acetate and then ether gave 1403g of the sodium salt, mp 296-300°C dec. Solution of the salt in 5400 ml of water, cooling to 10°C and acidifying with 1420 ml of glacial acetic acid gave a nearly solid mass. The product was collected and washed with water; weight 1244g (97.7%), mp 77.5-78°C. Recrystallization from methanol gave analytically pure material, mp 98-100°C.

3,4-Dimethoxyphenylacetone

Crude nitrile from above (1212 g) was added over one hour to a stirred and cooled (0-5°C) solution of 3008 ml of concentrated sulfuric acid in 727 ml of water. The brown solution resulting on heating to 80°C for 10 minutes was cooled to 0°C and diluted (stirring) with 10.75 L of cold water. During heating on the steam-bath for 3 hours, an oily layer separated. After cooling, ether extraction, washing of the extract with dilute bicarbonate solution, and drying, the ether extract was concentrated to give 610.8g (56.5%) of orange oil which was purified by vacuum distillation; weight 549.3g (51%), bp 142°C/2.6mmHg (bp 118°C/0.4mmHg).

alpha-(3-Methoxyphenyl)-acetoacetonitrile

Treatment of 3-methoxyphenylacetonitrile essentially as described above for 3,4-dimethoxyphenylacetorlitrile was followed by addition of petroleum ether to precipitate 275g of the sodium salt of alpha-(3-methoxyphenyl)-acetoacetonitrile, mp 280-290°C (dec). After acidification with acetic acid the yield was 206g (83.6%), mp 83-85°C. Two recrystallizations from methanol gave analytically pure material, mp 90-91°C.

3-Methoxyphenylacetone

The crude nitrile was hydrolyzed as described for 3,4-dimethoxyphenylacetonitrile  but at 90-95°C. The yield of ketone was rather poor (46.5%), and considerable unreacted nitrile was recovered. After redistillation the ketone was analytically pure, bp 95-97°C/0.7 mmHg.

Ref: JACS 77, 700 (1955)

Nemo Tenetur
The yield could be improved by using H3PO4 instead of H2SO4 for hydrolysis. I saw this in a short CA reference some years ago for plain, unsubstituted alpha-acetylbenzylcycanide, it should work also for substituted derivatives.

Osmium
This reaction sequence sucks, it's long, tedious and doesn't work nearly as well as it seems. I've done it before with straight benzyl cyanide, I can still smell that shit when I think back.

An alternative would be hydrolysing the benzylcyanide to PAA, and reacting with Ac2O and NaOAc. Much easier to do and yields won't be any worse.

Rhodium
Calcium acetate works just as well, and at the same time being cheaper and environmentally friendly:

https://www.rhodium.ws/chemistry/p2p.paa-ca.html

Osmium
The Vogel procedure for ketene production which will produce 1kg of acetic anhydride per day from cheap acetone and some electricity.

Osmium: "Re: Acetic Anhydride" (Acquisition Forum)
Osmium: "Re: Relativly simply Acetic Anhydride Synth." (Chemistry Discourse)

Since most of the anhydride can be reused, it doesn't matter when a big excess is used. Yields will definitely be better than the carboxylic salt pyrolysis. I tried that one too, with the barium salt, and I think the flask I used is still dirty many years later. Yield was horrible of course, a few percent of an ugly stinking mixture of all kinds of crap.
 
 
 
 
    Aurelius
(Hive Addict)
12-23-02 14:46
No 392001
      Epoxide Rearrangement to Ketone     

PolytheneSam
Epoxide to ketone using MgCl2

MgCl2 works as a  catalyst for converting epoxides to ketones.
from Mono-Olefins, Chemistry and Technology by F. Asinger 1968

see also
Patent US4731482

(text by aurelius from pic in original post)

“If other catalysts are used- such as, for example, magnesium halides- epoxides can be converted almost exclusively into methyl ketones.  Thus propylene oxide is about 90% converted into acetone [47].  Propylene Oxide can also be converted smoothly into propionaldehyde, so that, depending on the catalyst used, three different products can be obtained, each with a yield of about 90%. 


[47] H. Koch & H. Van Ray, Brennst.-Chemie 32, 161-74 (1951)
(brennstoff chemie: journal on combustibles and their refined products) PolytheneSam
It appears that 47 refers to DE 1119246 and Chem. Abstr. 57, 7176i (1962)
Patent DE1119246
British equivalent:
Patent GB905821
Similar procedure using K or Na iodide and PEG.  MDP2P seems to be listed in table 1. Patent US4734529
Osmium
They say that safrole-epoxide can be isomerised to MDP2P with NaI/PEG in 80-90% conversion. Yield of end product, at least for isosafrol epoxide is lower however. Maybe it could be improved
PolytheneSam
I wonder how if the intermediate in Patent US2243295 can be electrolytically reduced.  See Patent US3838027
Patent US3935272 olefins to epoxides and ketones
Rhodium
MgCl2 is much cheaper than LiI

3 propylene oxide with H3PO4а 1-propanol
same with MgCl2а acetone
same with ?* а propionaldehyde (* can probably be found in the book or the patent)

A process for the preparation  of allyl alcohol from propene glycol has been developed; allyl alcohol can also be obtained from actylene via propargyl alcohol by REPPE’S method, and from acrolein by reduction of the formyl group to the primary alcohol group.”

The Carlo Venturello patent above is the same as https://www.rhodium.ws/chemistry/peroxotungstate.html
 
 
 
 
    Aurelius
(Hive Addict)
12-23-02 15:02
No 392005
      Oxidation of alcohols with (NH4)2Cr4O7 in solution
(Rated as: excellent)
    

Oxidation of alcohols by (NH4)2Cr2O7 in solution and under solvent-free conditions
Shirini, F.; Zolfigol, M. A.; Mallakpour, B.; Mallakpour, S. E.; Hajipour, A. R.
Australian Journal of Chemistry  (2001), 54(6), 405-406 

Here they oxidize P2Pol to P2P in 10 minutes with a 90% yield. (Benzyl alcohol is also oxidized to benzaldehyde 5 min, 92% yield, solvent free, or 95% yield with solvent).

The oxidation of various alcohols was investigated using (NH4)2Cr2O7 in the presence of Mg(HSO4)2 and wet SiO2 (Scheme 1), at room temperature. Yields and reaction times are given in table 1. Over oxidation of the products, using this method, was not observed.

Scheme 1:
(i) (NH4)2Cr2O7/Mg(HSO4)2/wet SiO2, solvent free, room temperature.
(ii) (NH4)2Cr2O7/Mg(HSO4)2/wet SiO2, n-hexane, room temperature.
Substrate: 1-phenylpropan-2-ol
Product: phenylpropan-2-one
Solvent free oxidation (i): Time 10 min, yield 90%
Oxidation in solvent (ii): Time 300 min, yield (mixture)

The omission of the solvent not only eases the workup, but reduces the reaction time. This method is not suitable for the oxidation of allylic alcohols (e.g. cinnamyl alcohol).
It should be noted that the oxidation did not proceed using any of Mg(HSO4)2, ammonium dichromate or wet SiO2 alone. These results could be attributed to the in situ generation of H2CrO4 in low concentration at the surface of wet SiO2 by the solid inorganic salts (Mg(HSO4)2 and (NH4)2Cr2O7.
The authors note that this method “is attractive for large-scale operations”

Experimental:
Oxidation of Benzyl Alcohol to benzaldehyde under solvent free conditions: a typical procedure.

To a mixture of Mg(HSO4)2 (0.654 g, 3mmol), wet SiO2 (50% w/w, 0.1 g) and (NH4)2Cr2O7 (0.126 g, 0.5 mmol), was added benzyl alcohol (0.108 g, 1 mmol). The resultant mixture was shaken at room temperature for 3 minutes. The progress of the reaction was monitored by TLC. The reaction mixture was triturated with CH2Cl2 (10mL) and then filtered. Anhydrous MgSO4 was aadded to the filtrate and the mixture filtered after 10 min. Evaporation of the solvent followed by column chromatography on silica gel gave the benzaldehyde in 92% yield.
 
 
 
 
    Aurelius
(Hive Addict)
12-23-02 15:04
No 392006
      P2Pol to P2P     

Found by Chemistris

A mild and efficient oxidation of alcohols with N-tert-butylphenylsulfinimidoyl chloride in the coexistence of zinc oxide.
Matsuo, Jun-Ichi; Kitagawa, Hideo; Iida, Daisuke; Mukaiyama, Teruaki.    Department of Applied Chemistry, Faculty of Science,  Science University of Tokyo,  Tokyo,  Japan.   
Chemistry Letters  (2001),(2), 150-151.  

P2Pol is oxidized to P2P in 66% yields using N-tert-butylphenylsulfinimidoyl chloride in the presence of ZnO. The conditions for P2Pol to P2P are given as room temperature, 30 min.

Experimental (typical procedure):
To a stirred white suspension of 2 (2-phenylethanol, 70 mg, 0.0.57) and zinc oxide (233 mg, 2.86 mmol) in dry CH2Cl2 (1.5mL) was added a solution of 1 (N-tert-butylphenylsulfinimidoyl chloride, 185 mg, 0.86 mmol) in CH2Cl2 (2mL) at 0°C. The reaction mixture was stirred for 30 min at the same temperature and then quenched with water (5mL). The mixture was filtered through Celite and the filter cake was washed with CH2Cl2 and water. The layer were separated and the aqueous phase was extracted with CH2Cl2. The yield of 3 (phenylacetaldehyde, 0.52 mmol, 91%) was determined by GC analyses of the combined organic phase using an internal standard.

Table 1: Effect of bases
Entry   Base                Yield% (phenylacealdehyde)
3       MS4A (1g/mmol)      73 (56)b
6       MgO (10 eq.)        58
7       CaO (5 eq.)         56
8       BaO  (5 eq.)        70
12      ZnO (5eq)           91 (35)c  (0)d
b 3 g/mmol used, c ZnO (2 eq) used, d N-tert-butylphenylsulfinimidoyl chloride was not used
MS = molecular sieves
Notes:  in order to prevent decomposition of the formed carbonyl product, the oxidation using N-tert-butylphenylsulfinimidoyl chloride must be conducted under as neutral conditions as possible. Therefore, the original procedure was changed by replacing DBU with another suitable scavenger of HCl, which was liberated during the formation of an intermediate, alkoxysulfilimine, from N-tert-butylphenylsulfinimidoyl chloride and an alcohol. Then it was thought that an insoluble solid base would be suitable because it would not cause decomposition of the formed carbonyl  compounds.
 
 
 
 
    Aurelius
(Hive Addict)
12-23-02 15:07
No 392008
      P2Pol to P2P
(Rated as: good read)
    

Found by Chemistris

Oxidation process of alcohols using periodic acid and chromium catalyst.
Tschane, David M.; Song, Zhiguo; Zhao, Mangzu.  (Merck and Co., Inc., USA).    PCT Int. Appl.  (1999), 28 pp. 
WO9952850 Patent WO9952850
Also the equivalent:
“A Novel Chromium Trioxide Catalysed Oxidation of Primary Alcohols to the Carboxylic Acids”
Zhao, M. Li, J.  Song, z.  Desmond, R.  Tschaen D.M.  Grabowski E.J.J. and Reider P.J.
Tetrahedron letters 39 (1998) 5323 – 5326.
(also see Acme: "A new oxidation for MDP2-ol?" (Methods Discourse))
P2Pol is oxidized to P2P in 98% yields using periodic acid and chromium trioxide.
Also phenylethanol to phenylacetic acid (96%)


Since strong acids enhance the oxidation potential of CrO3, H2SO4 was added to the reaction mixture. This appeared to improve the oxidation slightly. On the other hand, water had a dramatic effect on the reaction rate. By eliminating water from the system, complete reation occurred in less than 15 minutes at room temperature. Subsequently, we found that the presence of small amounts of water attenuated the oxidation strength of the system and provided cleaner reactions. Thus the best yields can be obtained by adding a solution of H5IO6/CrO3 (2.5 equiv./1.1 mol % *) in wet MeCN (0.75 v % water) to the alcohols at 0 – 5 [deg] C. The reactions were typically complete within one hour. It should be noted that no reaction was observed in the absence of chromium trioxide. Substituting periodic acid with other oxidants (H2O2, t-BuO2H, AcO2H etc without TsOH) was unsuccessful. For unknown reasons the reaction also failed to give the desired product when carried out in acetone.
Typical procedure: A stock solution of H5IO6/CrO3 was prepared by dissolving of H5IO6 (11.4 g, 50 mmol) and CrO3 (23 mg, 1.2 mol %) in wet MeCN (0.75 v % water) to a volume of 114 mL (complete dissolution typically required 1-2 hours). The H5IO6/CrO3 solution (11.4 mL) was then added to a solution of the alcohol 1 (2.0 mmol) in wet MeCN (10 mL, 0.75 v water) in 30-60 minutes while maintaining the reaction temperature at 0-5 [deg]. The mixture was aged at 0°C for 0.5 h and the completion of the reaction was confirmed by HPLC assay. The reaction was quenched by adding an aqueous solution of Na2HPO4 (0.60 g in 10 mL water). Toluene (15 mL) was added and the organic layer was separated and washed with 1/1 brine/water mixture (2 x 10mL) then a mixture of aqueous NaHSO3 (0.22 g in 5 mL water) and finally brine (5 mL). The organic layer was then concentrated to give the crude carboxylic acid 2. Most of the crude products were quite pure based on 1H NMR and HPLC assay.“
* Only 1.25 equivalents of periodic acid and 0.6 mol % CrO3 are required for secondary alcohol to ketone.

Substrate: P2Pol
Product: P2P
Temp/H5IO6/CrO3: 0/1.25/0.6
Yield: 98%

Substrate: Phenethanol
Product: phenylacetic acid
Temp/H5IO6/CrO3: 0/2.5/1.1
Yield: 96%
 
 
 
 
    Aurelius
(Hive Addict)
12-23-02 15:10
No 392009
      biochemical oxidation P2Pol to P2P     

Found by Chemistris


Stereoselective oxidation and reduction by immobilized Geotrichum candidum in an organic solvent. 
Nakamura, Kaoru; Inoue, Yuko; Matsuda, Tomoko; Misawa, Ibuki.    Institute for Chemical Research,  Kyoto University,  Uji, Kyoto,  Japan.   
Journal of the Chemical Society, Perkin Transactions 1: Organic and Bio-Organic Chemistry  (1999),(16), 2397-2402. 

P2P is formed in 44% yield from P2Pol

Abstract
Cells of the fungus, Geotrichum candidum, were immobilized on a water-absorbing polymer and used for stereoselective oxidn. and redn. in an org. solvent using cyclohexanone, cyclopentanol or 2-alkanols as additive.  Enantiomerically pure (R)-1-arylethanols were obtained by the stereoselective oxidn. of racemic 1-arylethanols, whereas enantiomerically pure (S )-1-arylethanols were obtained by the redn. of the corresponding ketones, in contrast to redn. in water by the free cells in which (R)- or (S )-1-arylethanols were produced in low ee.  The reaction mechanism was investigated by measuring the partition of the substrates and products between the org. phase and aq. phase in the polymer around which the cells were immobilized.  Deuterated compds. were used to det. the role of the additives.
 
 
 
 
    Aurelius
(Hive Addict)
12-23-02 15:12
No 392011
      TrifluoromethylP2Pol to TrifluromethylP2P     

Found by Chemistris



Homochiral (R)- and (S)-1-heteroaryl- and 1-aryl-2-propanols via microbial redox.
Fogagnolo, Marco; Giovannini, Pier Paolo; Guerrini, Alessandra; Medici, Alessandro; Pedrini, Paola; Colombi, Nicola.  Dipartimento di Chimica,  Universita di Ferrara,  Ferrara,  Italy.
Tetrahedron: Asymmetry  (1998), 9(13), 2317-2327. 

Trifluoromethyl-P2P is formed from trifluoromethyl-P2Pol in 43% yields

Abstract:
Prepn. of various heteroaryl propanols and of the corresponding propanones as starting materials for microbial redox was described.  The kinetic resoln. of the racemic propanols was achieved via oxidn. with Pseudomonas paucimobilis and Bacillus stearothermophilus [(R)-alcs., ee 74-100%].  Similar results are achieved with 3-(2-hydroxypropyl)trifluoromethylbenzene.  The redn. of the propanones with baker's yeast and other fungi gave (S)-alcs. (ee 100%).  The pure (S)-alcs. were also obtained by redn. of 1-[3-(trifluoromethyl)phenyl]-2-propanone.  1-[(4,4-Dimethyl)-2-(D2)oxazolinyl]-2-propanone and 1[2-(D2)-thiazolinyl]-2-propanone were not reduced.803.  The heterocyclic rings of (S)-5-(2-hydroxypropyl)-3-methylisoxazole and of (S)-2-(2-hydroxypropyl)-4-methylthiazole were deblocked to a homochiral enamino ketone (78%) and to a protected b-hydroxy aldehyde (73%), resp.  Also, (R)-3-(2-hydroxypropyl)trifluoromethylbenzene was transformed into a homochiral precursor of (S)-fenfluramine (overall yield 65%).
 
 
 
 
    Aurelius
(Hive Addict)
12-23-02 15:15
No 392012
      P2Pol to P2P     

found by Chemistris

Oxidation of alcohols by silica gel-supported bis(trimethylsilyl) chromate.
Lee, Jong Gun; Lee, Jung A.; Sohn, Soo Yun.  College Natural Sciences,  Pusan National Univ.,  Pusan,  S. Korea.   
Synthetic Communications  (1996), 26(3), 543-9. 

P2P is formed in 91% yield from P2Pol

Chromium oxidants absorbed on solid supports such as PCC on alumina, chromic acid on silica and chromyl chloride on silica-alumina are reported as giving better yields under milder conditions, than the corresponding parent oxidants. Herein we report that BTSC (bis(trimethylsilyl) chromate) supported on chromatographic grade silica gel is very efficient for the oxidation of alcohols of various types to the corresponding carbonyl compounds.  Heating chromic anhydride with a slight excess of hexamethyldisiloxane in carbon tetrachloride produces a nearly homogenous solution. BTSC supported on silica gel was conveniently prepared simply by adding silica gel into the vigorously stirred pre-made solution. The dark brown free-flowing solid was obtained on evaporation of volatile materials. It can be stored in a dark brown bottle without appreciable loss of reactivity for at least 3 months.
The oxidation reactions were carried out simply by adding  1 – 2 equivalents of BTSC supported on silica gel to the stirring solution of an alcohol in CH2Cl2. [Yields are significantly higher than reactions not supported]

Alcohol: 1-phenyl-2-propanol
Temperature: 40°C
Time: 30 min.
Product: phenylacetone
Yield: 91%

Alcohol: cyclohexanol
Temperature: 25°C
Time: 15 min.
Product: cyclohexanone
Yield: 99%


Preparation of bis(trimethylsilyl) chromate supported on silica gel: Into a solution of hexamethyldidiloxane (64mL, 0.3 mol) in 200mL  of carbon tetrachloride was added chromic anhydride (30g, 0.3 mol). The reaction mixture was stirred in a 50°C bath for 5 hours. Solid chromic anhydride dissolved and the dark red mixture became a homogenous solution. Silica gel (130 g) pre-dried in a 120°C oven overnight was added into the warm reaction mixture and the resulting mixture was stirred further for 5 hours. The solvent and other volatile components were distilled off under reduced pressure to afford 193 g of supported chromium oxidant. The calculated 1.54 mmol per gram of this typical batch of the reagent was regarded as the effective chromium concentration.

Experimental:
Oxidation of cyclohexanone: Silica gel suppored bis(trimethylsilyl) chromate (0.8 g, equivalent to 1.2 mmol of chromium (VI) oxidant) was placed in a round bottem flask and covered with 5mL of CH2Cl2. Into the stirring mixture was added cyclohexanol (105 mg, 1.04 mmol). After 10 minutes of stirring the reaction mixture was gravity filtered and washed with 10mL of ether or CH2Cl2. Evaporation of the solvent gave 101 mg of cyclohexanone (99%) of which GC analysis showed negligible amounts of impurities.
 
 
 
 
    Aurelius
(Hive Addict)
12-23-02 15:18
No 392014
      P2Pol to P2P     

found by Chemistris

Asymmetric synthesis of (S)-4,5-difluoro-2-methylindoline.
Tsuji, Koichi; Ishikawa, Hiroshi. Microbiological Res. Inst.,  Otsuka Pharm. Co. Ltd.,  Tokushima,  Japan.   
Synthetic Communications  (1994), 24(20), 2943-53. 

Ketone from alcohol: CrO3/H2SO4 99% yield.

N-[(S)-N-p-Tolylsulfonyl]prolinyl-2-(2-oxo)propyl-3,4-difluoroanilide  ; (4a):
A solution of CrO3 (0.75g, 7.5 mmol) in 30% aqueous H2SO4 (2mL) was added to a stirred solution of the anilide 3a (3.95 g, 0.009 mol) in acetone (36mL) and the resulting mixture was stirred 45 min at room temperature. After addition of i-PrOH (0.84 mL, 0.011 mol), the mixture was filtered and the filtrate was evaporated. The residue was dissolved in a mixture of CH2Cl2 and water. The aqueous phase was extracted twice with CH2Cl2 and the combined organic extracts were washed twice with water, then dried over MgSO4. Evaporation of the solvent afforded ketone 4a (3.89 g, 99%) as a brown solid, which was used in the next step without further purification
 
 
 
 
    pHarmacist
(Hive Bee)
12-23-02 15:21
No 392015
User Picture 
      Excuse me for interrupting     

found by Chemtris allover... I did the same misstake once. I'm telling you this due to TFSE, if someone gets interested in finding other posts by the poster ChemisTris.

His/Her name is ChemisTris.

"Turn on, Tune in and Drop Out"
 
 
 
 
    Aurelius
(Hive Addict)
12-23-02 15:22
No 392016
      Synth of P2Pol     

Found by Rhodium

Phenyl-2-propanol
Nice! For the synthesis of P2Pol without the use of Hg, these may be worth looking into:

* Bull. Chem. Soc. Jpn. (1983),  56(4),  1089-94. (In English)
  1-phenyl-2-propanol from propylene oxide and benzene.

* Patent US 5223633
  Complex method of making 1-phenyl-2-propanol which is probably not useful.

* Grignard reactions of phenyloxirane in the presence of titanium tetraisopropoxide.    
  Zh. Org. Khim.  (1992),  28(7),  1377-9.
  65-80% P2Pol from Styrene epoxide

* 1-Phenyl-2-propanol from phenylmagnesium bromide and propylene oxide
  JACS 62, 2295-2300 (1940)

To a Grignard reagent prepared from 471g of bromobenzene and 74g of magnesium in 1400 cc. of ether as slowly added 180g of propylene oxide. After refluxing for 20h, the mixture was treated with ammonium chloride solution. The crude product (bp 90-110°C/14 mm), amounted to 280g. On fractionation there was obtained 244g (60%) of 1-Phenyl-2-propanol, bp 105.5-107.0°C/14-15mmHg.
 
 
 
 
    Aurelius
(Hive Addict)
12-23-02 15:33
No 392019
      P2P     

Idea by PrimoPyro

Crazy Elegant Phenyl-2-Propanone Synthesis
(aurelius was not sure if the pic would copy to compilation, so here’s the post number if you want the pic- post 363485)

Question: Is alpha-haloethyl acetate stable? CH3-CH(X)-OAc.

Why?

Alpha-haloethyl acetate (if can be used to get P2P enol acetate ester.  Because it's the only reasonable way I can think of to get P2P enol acetate ester as a precursor. We can do that with a Wittig Reaction with benzaldehyde of course. March's 5th claims that in most cases, COOR groups are inert to Ylides/phosphoranes.

Why would you want P2P enol acetate? Well, the obvious hydrolysis of course to P2P, but take a look at this instead;

(insert pic here)

First we make our P2P enol acetate from the Wittig. Then we take benzyl chloride in ether and magnesium, to form benzylmagnesium chloride-ether complex, a Grignard Reagent. This Grignard Reagent reacts with the acetate function of our enol ester to give the compound pictured on the middle of the right column.  Hydroysis of the salt forms P2P. Well, in aqueous acidic solution, our two products formed by the decomposition of the Grignard adduct, both form P2P! The enol tautomerizes to phenyl-2-propanone, where the gem-diol loses H2O to form a ketone: phenyl-2-propanone. I find this quite humorous.

Realistically speaking though, all I really wanted to say was that ethyl acetate can be used instead of acetonitrile in the Grignard Reaction of benzyl chloride --> P2P. Ethyl acetate is a lot easier for most to get than acetonitrile, and is cheaper too

Sometimes the obvious escapes me.

No need for a Wittig. Use chloroethenyl acetate instead, and two moles of phenylmagnesium chloride.

The first mole replaces the chlorine on the ethenyl group, forming the P2P enol acetate in situ, then the second furthers the reaction like normal.

So......is Cl-CH=CH-OAc stable?

Rhodium
Here is a reference for half of your reaction: https://www.rhodium.ws/chemistry/phenylacetone.html#ylide

PrimoPyro
Yeah, I had gone back to that reference when I got home from work yesterday. I had the idea/question while spacing out at work lol.
 
 
It should also be mentioned, for those not familiar with these type reactions, that the inverse of the first reaction, the reaction of benzyl chloride with ethyl acetate, can be inverted and still achieve the exact same product. If you have phenylacetic acid, and don't have acetic anhydride, or lead acetate, etc, you can react an ester of your phenylacetic acid with methyl magnesium iodide (the Grignard Adduct of methyl iodide) to get phenyl-2-propanone on hydrolysis.
 
 
 
 
 
    Aurelius
(Hive Addict)
12-23-02 15:39
No 392022
      Suggestion on P2P     

by Slappy (this wouldn't even have been put in here, but Slappy is well known)

Novel High yield P2P synth
I actually have a high yielding, one step synthesis of P2P and Substituted P2P's starting from Bromobenzene or substituted Halobenzene and Ethyl Acetoacetate, that I have been working on for some time now, but have not had time to share. Details will follow shortly I presume.

The only hint I'll give is that it is a Pd-catalysed ketone [alpha]-arylation
 
 
 
 
    Aurelius
(Hive Addict)
12-23-02 15:51
No 392026
      allylbenzenes to alcohols to P2Ps     

found by Poix

Propanones from Allylbenzenes through Propanols
Although 1-aryl-2-propanones are commonly prepared from phenols by formylation, modified Knoevenagel condensation with nitroethane and subsequent reduction and hydrolysis, we employed the Claisen rearrangement to introduce the three-carbon unit both for the efficiency and simplicity of operation. Hydration of the double bond was then accomplished in high yield by oxymercuration-reduction after due protection of the phenolic group. Oxidation of the resulting alcohol to the corresponding ketone was unsatisfactory with commonly used reagents. However, it could be successfully carried out using Na2Cr2O7-H2SO4 in DMSO, analogous to that of 2-phenyl ethanol. This procedure thus offers a convenient alternative to the preparation of 1-aryl-2-propanones, the synthetically useful intermediates.   

3-allyl-4-methoxy-toluene:
A mixture of o-allyl-p-cresol (13g. 87 mmol), dimethyl sulphate (18.6g 148 mmol) and 10% aq. NaOH (50 ml) was stirred at 100°C for 4h. Usual work up and distillation in vacuo gave 12g (84% of the product bp 82°C 5mm Hg)

1-(2-Methoxy-5-methyl)-phenyl propan-2-ol:
3-allyl-4-methoxy-toluene (9.7 g 60 mmol) was added dropwise into a stirred solution of Hg (OAc)2 (18.6 g 60 mmol) in aq. THF (1:1, 120 ml) at room temperature. After 20 min 3 M NaOH (60 ml) was added slowly with cooling followed by a solution of 0.5M NaBH4 in 3 M NaOH (60 ml). Filtration, usual work up and distillation afforded the product 2 as a colourless liquid (8.5 g, 79%), bp 123°C (10 mm Hg), which solidified on standing.

1-(2-Methoxy-5-methyl)-phenyl propan-2-one:
Sodium dichromate (6.6 g, 16.2 mmol) was dissolved by warming in 60g (54.5 ml) of DMSO containing 5.4g (30 mmol) of 1-(2-Methoxy-5-methyl)-phenyl propan-2-ol. The solution was cooled, conc. H2SO4 (4.5 ml) was added dropwise and the mixture heated at 70°C for 45 min. The resulting green solution was cooled, poured into crushed ice and extracted with ether. Work up and distillation yielded 4.67g (87%) of 1-(2-Methoxy-5-methyl)-phenyl propan-2-one, bp 130°C (10 mm Hg)

Same procedure on 2,5-Dimethoxyallylbenzene https://www.rhodium.ws/chemistry/25.meo.allylbenzene.html will yield the 2,5-diMeo-P2P

source:Tet. vol 41 p 107-110 1985
 
 
 
 
    Aurelius
(Hive Addict)
12-23-02 16:07
No 392031
      P2P from phenyl sodium and ethyl acetate     

found by Mountain Girl

P2P from phenyl sodium & ethyl acetate
Another of the 'wanted references':
Tsuruta, Bull. Inst. Chem. Research, Kyoto Univ., vol. 31, 190-200 (1953) (in English); Chemical Abstracts vol 49, 6183b

It seems that there was a chem abstract mistranslation. The synth actually uses chlorobenzene not benzyl chloride.
The paper is entitled 'Michael and some related reactions' and is about 10 pages long. It's not practical to scan and post so I will just give the most important experimental section:
 
ii. Ethyl acetate and benzyl sodium as reactants.
Benzyl sodium was prepared according to Gilman's method*. Phenyl sodium prepared from 34 g (0.3 mol) of chlorobenzene and 13.8 g (0.6 atom) of sodium sand in 150 ml of dry toluene was refluxed on an oil bath for 3 hrs until the black colour of the precipitates turned to brownish black through reddish brown.
The mixture was cooled to -4°C and was added 26.5 g (0.3 mol) of ethyl acetate in toluene in the course of 5 minutes under vigorous stirring. After 2 hrs stirring at about 0°C, the mixture was allowed to stand overnight at room temp. and was added some methanol to dissolve the unreacted sodium present, acidified with aqueous acetic acid and the aqueous layer shaken with toluene. The toluene residue was fractionated at 17 mm.

            b.p.(17 mm) 130 - 155 °C    7 g  (VIII)
            b.p.(17 mm) 195 - 200 °C    7 g  (IX)

An oil (VIII) readily crystallized by ice-cooling and reacted with semicarbazide to give crystals which melted at 187-188°C (recrystallized from alcohol-benzene mixture).
Mixed melting point measurement with an authentic phenyl acetone semicarbazone assured (VIII) was phenyl acetone. The viscous oil (IX) was not distinctly identified, but it is very likely that (IX) is methyl dibenzyl carbinol.

* H. Gilman, H. A. Pacevitz & O. Baine : J. Am. Chem. Soc, 62, 1514(1940)
 
 
 
 
    Aurelius
(Hive Addict)
12-23-02 16:12
No 392035
      New organoZinc synth of P2P     

Translated by Antoncho
originally written/found?  by Zealot in the Russian HyperLab

Zealot: a new zincorganic synthesis of P2P's
The following is a verbatim translation of zealot: "Синтез Р2Р через цинкорганику." (Russian HyperLab):


 


Zincorganic synthesis of P2P's.

2C6H5I                  +                2Zn                  =              (C6H5)2Zn              + ZnI2

M=204                                     M=65                                M=219
m=408 g                                  m=130 g                            m=219 g
m=63 g                                    m=22  g                             m=34 g
D=1,83 g/ml
V=34,5 ml

(С6H5)2Zn         +           2BrCH2C(O)CH3      =       2C6H5CH2C(O)CH3    +    ZnBr2

M=219                             M=137                                 M=134
m=219 g                           m=274 g                              m=268 g
m=30 g                             m=37,5 g                             m=37 g
                                         D=1,6 g/ml
                                         V=24 ml





Almost forgotten until recently, zincorganic comp'ds beecome now increasingly often used in synthesis once again. They are less reactive than Grignars, but in many cases - e.g., when it is necesary to preserve a functional group, this is an advantage rather than a drawback. For magnesiumorganics the more stable form is the asymmetric one, which exists in form of associate comp'ds (etherates); between the two  forms, RMgX and R2Mg, there exists a Schlenk (sp?) equilibrium. OTOH, for zincorganics the more stable is the symmetric form, which is conveniently obtained from the asymmetric one by heating. In addition to that, R2Zn species aren't prone to association and have a linear structure.

It remains a mystery for me as to why the well-known sources are full of methods making use of
cadmium-organics, while everyone knows of their extreme toxicity; beesides, the only way of obtaining those is thru exchange between Cd salts and Grignars, whilst Zn-organic compds can bee obtained directly from alkyl or aryl halides and are non-toxic. The traditional method of making them is from alkyl halides and Zn dust or foil. Later it was shown that alkyl bromides may bee used in this rxn as well as iodides, and only for the less active phenyl halides, iododerivatives must bee used.

The solvts employed in this rxn may bee ethers (diisopropyl, dibutyl) , toluene, benzene, xylols.


Synthesis:

20g Zn dust is washed w/5% HCl, then 2x100mls water, 2x50mls acetone and 2x50mls ether (Zealot keeps his ethers over KOH, which is reported to give even better results that traditional Na drying - proc. has been posted here). All is placed in a 500ml RBF w/a reflux condenser and 75 mls toluene is added. A mag stirrer is turned on, there's added several iodine crystals, and the system is flushed with Ar. Thru a dropping funnel there's added a little of the mixture of 75mls toluene and 34,5mls iodobenzene and the flask's bottom is heated to 80-90 C on oilbath. A vigorous reaction takes place, and if the condenser begins to choke up, heating and addition is disontinued for a while. A following nuance should bee noted: if in preparations of Grignars the faster the reagent is prepared, the better it is; with Zn comp'ds, after all the PhI has been added, the mixtr needs to bee boiled for additional 3-4 hrs to allow full disproportionation to ZnPh2. This is accompanied by heavy precipitation of ZnI. Thus obtained solution of diphenylzinc is as quickly as possible decanted from the precipitate, the latter is washed w/2x50mls toluene and all is placed into a dripping funnel.

The mixtr is gradually added to 24mls bromoacetone (see Antoncho: "Bromoacetone Synthesis" (Novel Discourse) for the best synthesis of bromoacetone, also by Zealot) in 50mls toluene, ZnBr2 precipitation occurs. The liquid phase is decanted and the solvent is evap'd , the residual oil is dissolved in 80mls MeOH and 20mls 40% aq. alkali and refluxed ~15 mins until the complete disappearance of lacrymatory bromoacetone smell. MeOH is removed and P2P is xtracted with ether.

Yields are usually around 70-80%.

The reaction also works well for preparing substituted phenylacetones.


A necessary note on the so-much-dreaded lacrymogenicity of bromoacetone: there were times when i successfully worked w/it at home (wearing an ordinary gas mask)

The reaction is moisture-sensitive, but there's a pleasant advantage over Grignars: it is carried in non-etheric solvts which can bee dried by simple distillation. Personally i use chromatographic grade solvts - they don't need to bee dried at all.
The principal concern is to wash and dry the Zn dust very thoroughly; if toluene gets turbid after its addition, that's a sure sign that there'll bee problems with starting the rxn and/or the yield.

This has been successfully performed on: p-iodoanisole, p-iodotoluene, p-iodobenzodioxole, iodomesitylene - the last one gave the lowest yield and the resulting amine was completely inactive.
 
 
 
 
    Aurelius
(Hive Addict)
12-23-02 16:20
No 392037
      Indolic P2P's (Indole 3-acetone)     

Found by Barium

Indole-3-acetone from indole-3-carboxaldehyde
(P2P related)
After quite some success with the Darzen´s condensation making phenylacetones, I decided to challenge some indole-3-carboxaldehyde to see what this puppy was good for.

synthesis:
Indole-3-carboxaldehyde, 14.5g (100mmol), ethyl 2-chloropropionate, 19.1g (140mmol), 25ml toluene and 30ml MeOH was added to a 500ml rb three-necked flask equipped with a thermometer and a magnetic stirbar. To this suspension 8.1g (150mmol) sodium methoxide (dissolved in MeOH) was added dropwise during 1 hour. This reaction was actually endothermic so no cooling was needed. After the addition was complete the reaction mixture was allowed to stir at room temp for two hours. During this time the substrate gradually dissolves and gives a orange solution from which a white precipitate separates.

10g (250mmol) Sodium hydroxide dissolved in 50ml water was then added dropwise during 10min. No cooling was applied here so the temperature reached 45 deg C. An additional 100ml water was added and the solution was stirred at 60 deg C for one hour. The toluene layer was separated from the aqueous layer (now deep red). Conc hydrochloric acid was added dropwise to the hot water solution until pH 3.5 was reached. A slightly yellowish crystalline mass fell out as the solution went acidic.  The acidic suspension was stirred at 80 deg C on a water bath for one hour to complete the decarboxylation. The mixture was extracted with 8x50ml ether and the combined organic layers washed twice with 200ml 10% sodium bicarbonate solution, twice with water and finally once with brine. The solvent was removed with a rotovap leaving slightly yellowish crystals.

Yield: 15g (87%) indole-3-acetone

I will recrystallise this and check the mp and report.

Recrystallization:
The crystals were suspended in 100ml boiling IPA, 60ml was removed by distillation and the remaining suspension was cooled to room temp and then chilled to 0 deg C for one hour. The crystals was removed by filtration and dried under vacuum at 60 deg C.

Yield: 14.1g indole-3-acetone as white crystals with a mp of 114-115.5 deg C.

Ethyl 2-chloropropionate is cheaper

Ethyl 2-chloropropionate is cheaper and, in my opinion, better to use than the methyl ester.
It has a lower vapor pressure and hence, stinks less.
I buy it from a major chem supplier for less than $100/L. In bulk it´s really cheap.
It is not bad at all to work with. Non-watched until now I guess, or very soon.

In some of the articles is quickly scanned trough before I started with this route
I remember seeing something about the chloro being better than the bromo or iodo.
This was due to some strange side reactions which could occur wiht the heavier haloesters.
But under the mild reaction conditions I´ve had I don´t see any problems with the bromoester.

From Vogel 5:th ed.

Ethyl 2-bromopropionate

In a 1L two-necked round-bottomed flask, equipped with a dropping funnel and a double surface reflux condenser to which is attached a gas absorbation trap, place 220g (135ml, 1.86mol) of redistilled thionyl chloride, and heat to boiling. Add 125g (126ml, 1.69mol) of pure propanoic acid at such rate that the mixture refluxes gently (1 hour). Reflux the mixture for a further 30 minutes to expel the dissolved sulphur dioxide, allow to cool and add 0.5g purified red phosphorus. Introduce 310g (100ml, 1.93mol) of dry bromine during 5-7 hours to the gently boiling propanoyl chloride, and then reflux the mixture for 7 hours, by which time the evolution of hydrogen bromide almost ceases.
Add the crude 2-bromopropanoyl chloride during 2 hours to 250ml absolute ethanol (note 1) contained in a three-necked round-bottomed flask, equipped with a mechanical stirrer and a reflux condenser. Complete the reaction by heating on a water bath for 4 hours, when hydrogen chloride is slowly evolved. Filter the reraction liquid into 500ml distilled water, separate the oil and wash it successively with water, sodium bicarbonate solution and water. Dry over calcium sulphate and distill at normal pressure to remove the low boiling fraction (largely ethyl bromide: 75g) and the at diminished pressure. Collect the ethyl 2-bromopropionate as a colourless liquid at 69-70 deg C @ 25mm Hg; the yield is 221g (72%)

Note 1. Substitute for methanol or another alcohol if you´d like.


it´s a novel route to a whole shitload of tryptamines

Indole-3-carboxaldehyde and ethyl 2-chloropropionate gives indole-3-acetone.
From indole-3-acetone and
ammonia                      alpha-methyltryptamine
methylamine              alpha,N-dimethyltryptamine
ethylamine                N-ethyl-alpha-methyltryptamine
propylamine              alpha-methyl-N-propyltryptamine
isopropylamine        N-isopropyl-alpha-methyltryptamine
allylamine           N-allyl-alpha-methyltryptamine
dimethylamine         alpha,N,N-trimethyltryptamine
diethylamine          N,N-diethyl-alpha-methyltryptamine
dipropylamine      N,N-dipropyl-alpha-methyltryptamine
diisopropylamine N,N-diisopropyl-alpha-methyltryptamine
diallylamine     N,N-diallyl-alpha-methyltryptamine

Indole-3-carboxaldehyde and ethyl chloroacetate gives indole-3-acetaldehyde
From indole-3-acetaldehyde and
ammonia                         tryptamine
methylamine             N-methyltryptamine
ethylamine               N-ethyltryptamine
propylamine             N-propyltryptamine
isopropylamine       N-isopropyltryptamine
allylamine               N-allyltryptamine
dimethylamine       N-N-dimethyltryptamine
diethylamine         N,N-diethyltryptamine
dipropylamine       N,N-dipropyltryptamine
diisopropylamine N,N-diisopropyltryptamine
diallylamine        N,N-dilallyltryptamine

5-methoxyindole-3-carboxaldehyde......I´m sure you get the picture by now.

If you animate with one or the other chiral alphamethylbenzylamine, and deprotect, as per [b]JMC 1986, 29, 10, 2014[b] both AMT stereoisomers would be available, pure, although this I would not advise, unless you have a license, and know how to do it properly, and have a signed note from your Mum.
 
 
 
 
    Aurelius
(Hive Addict)
12-23-02 16:23
No 392039
      Hydrocinnamic acid to P2P     

Idea by Rhodium (Go Rhodium!!)

Hydrocinnamic acid to P2P - byzantine or real?
3-Phenylpropionic acid (Hydrocinnamic acid) can be made in many ways (or bought for about $100/kg), and by subjecting it to Hell-Vollhardt-Zelinski reaction conditions (PBr3/Br2) you'll afford 2-Bromo-3-Phenylpropionic acid, which in turn can be reduced to 2-bromo-3-Phenyl-1-propanol (allylbenzene bromohydrin) using for example NaBH4 together with I2 or H2SO4.

By refluxing this bromohydrin lightly in dilute NaOH it will cyclize through an intramolecular Williamson etherification to form 2-benzyloxirane (allylbenzene epoxide), which finally is refluxed with LiI in EtOAc for a few hours to effect the isomerization of the epoxide to its methyl ketone; Phenyl-2-Propanone.
 
 
 
 
    Aurelius
(Hive Addict)
12-23-02 16:27
No 392041
      Found by Yellium P2Pol synth Taken from JACS ...     

Found by Yellium

P2Pol synth
Taken from JACS 2295, 62, 1940



1-phenyl-2-propanol

To a grignard reagent prepared from 471 g. of bromobenzene and 74 g. of
magnesium in 1400 cc. of ether was slowly added 180 g of propylene oxide.
After refluxing for twenty hours, the mixture was treated with aqeous ammonium
chloride solution. The crude product, b.p. 90-110 at 4mm, amounted to 280
g. On fractionation in the above-mentioned column there was obtained 244 g.
(60%) of 1-phenyl-2-propanol, b.p. 105-107 ' at 14-15 mm, n20d 1.5196. The
phenyl urethan melted at 88.2-88.8'.


1-phenyl-2-bromopropane

To a solution of 180 g. of phosporus tribromide in 100 cc. of benzen  was
added with cooling 128 g. of 1-phenyl-2-propanol during one hour. The
mixture stood at room temperature for two hours and was then slowly heated
for ten mintes. The benzene layer was washed with water and dried over
calcium chloride. The yield of twice distilled 1-phenyl-2-bromopropane,
b.p. 112.5-114.0 at 20-21 mm, n20d 1.5416 was 118 g. (63%). The same
bromide was obtained in 60% yield by refluxing 1-phenyl-2-propanol
with an excess of 48% hydrobromic acid.


note by Karel

If you add catalytic amount of cupprous salt to Grignard reagent before adding propylene oxide, you would afford better yields (i.e. Gazz.Chim.Ital. 1991, 455 - 93% yield m-OMOM-P2Pol).
 
 
 
 
    Aurelius
(Hive Addict)
12-23-02 16:43
No 392046
      2,5-dimeobenzaldehyde to 2,5-dimeoP2P     

Idea and synthesis work by barium

Synthesis of 2,5-dimethyoxyphenylacetone:
To a 500ml rb three-neck flask with a thermometer and a stir bar was added 16,6g (100 mmol) 2,5-dimethoxybenzaldehyde and 18,3g (150mmol) methyl 2-chloropropionate. 100ml MeOH was added to get the benzaldehyde into solution. While keeping the reaction mixture at 15 deg C, 8,1g (150 mmol) sodium methoxide in 50ml MeOH was added dropwise during 40min. When all alkoxide was added the reaction mixture was allowed to come to room temp and stir for another hour. The reaction mixture was then added to 10g (250 mmol) NaOH in 40ml water while keeping the temp at 20 deg C, then all was stirred at room temp over night. The next morning there was a thick white precipitate in the solution. To this 15% aq HCl was added until pH 3,5 was reached. This caused a clear yellow oil to fall out. The solution was heated on a waterbath at 65 deg C for two hours to complete the decarboxylation. The methanol was removed by distillation in a rotovap and the ketone was isolated by steam distillation. The distillate was extracted with 3x75ml toluene and the collected toluene phases dried over MgSO4. The toluene was removed by distillation in a rotovap leaving a clear yellow oil.

Yield: 14.94g (76,9 mmol) 2,5-dimethoxyphenylacetone
Purity: 96% by HPLC


This can and will be improved, but it was just a first trial.
There has been a successful reaction run of this first reaction run at the 500mmol level (5x larger than the scheme above)Yield: 91.5g (94.2%) 2,5-diMeOphenylacetone as a yellow oil which turns orange on several hours exposure to air.

Synthesis of 2,5-dimethoxyphenylacetone; a variation on a theme: Aq. NaOH/Aliquat 336 instead of NaOCH3

16,6g (100 mmol) 2,5-dimethoxybenzaldehyde was added to 50ml toluene and 2g (approx 5 mol%)Aliquat 336 in a 250ml rb flask with a thermometer and a stirbar, then 5,6g (140 mmol) NaOH was dissolved in 7ml water and added to the flask. To this two-phase system 14,7g (120 mmol) methyl 2-chloropropionate was added dropwise during 30 minutes. The temperature was maintained at 20 deg C using a icebath. When the addition was complete the reaction mixture was stirred for another 30 minutes at roomtemp. 10g (250 mmol) NaOH dissolved in 50ml water was then added and the mixture was stirred for another hour at room temp. The phases was then separated and the aqueous layer saved. The toluene layer was extracted with another 50ml water and then discarged. The combined aqueous phaes was then acidified with dilute HCl until pH 3 was reached.
The lovely yellow oil comes once again....

The yield from the trial was 5.4g ketone. Not too shabby considering the very short reaction time and overall impatience.
 
 
 
 
    Aurelius
(Hive Addict)
12-23-02 16:45
No 392047
      Mechanism of the Darzen Condensation     

Mechanism explained by Rhodium

Mechanism of the Darzen Condensation as used in Barium's Benzaldehydes to Phenylacetones:

The base grabs the halogen from the alpha-halo ester, forming a carbanion, which adds to the carbonyl, forming the epoxide. The reaction is a Darzen Condensation, and you should be able to look up the detailed mechanism either online or at the library.

The epoxy ester is then hydrolyzed and the formed acid decarboxylated. During the decarboxylation, the epoxide also rearranges to the ketone
 
 
 
 
    Aurelius
(Hive Addict)
12-23-02 16:48
No 392048
      2,4-diMeOphenylacetone by darzen condensation     

Idea and Synthesis work by Barium

2,4-dimethoxyphenylacetone

Synthesis:

Not optimized but pretty good.

100mmol 2,4-dimethoxybenzaldehyde, 120mmol NaOH, 2g aliquat 336, 50ml MeOH and 50ml toluene was stirred together in a 500ml rb flask. 120mmol methyl 2-chloropropionate was added dropwise over 20 min while the temp was kept at 20 deg C. When addition was complete stirring was maintained for another 30 min. 150mmol NaOH in 15ml water was then added and stirring was continued for 30 min.
Workup was done as before but without steam distillation leaving 6,7g 2,4-dimethoxyphenylacetone as a bright yellow oil.

Reaction times should be longer to give optimum yields
 
 
 
 
    Aurelius
(Hive Addict)
12-23-02 16:51
No 392051
      Various novel phenylacetones and yeilds     

idea and synthesis work by Barium

Synthesis of several novel phenylacetones by means of a Darzen Condensation
This morning I decided to try a few different benzaldehydes just to see if they would give the corresponding phenylacetones as well. The reactions were not allowed to run full time.

100mmol 2-fluorobenzaldehyde 34% ketone 
100mmol 4-fluorobenzaldehyde 38% ketone 
100mmol 2,4-dimethoxybenzaldehyde 31% ketone 
100mmol 3,4-ethylenedioxybenzaldehyde 42% ketone


Reaction conditions used were:
100mmol aldehyde, 2g aliquat 336, 120mmol NaOH as a 50% aq. soln. and 50ml toluene was stirred at 20 deg C. 105mmol methyl 2-chloropropionate added dropwise during 30 min while temp was kept at 20 deg C. Stirring was continued for 30 min then 150mmol NaOH in 50ml water was added and stirring continued for 1 hour. Phases separated and toulene phase extracted once with 50 ml water. The combined aqueous extracts was acidified to pH 3,5 with diluted HCl, heated to 45 deg C for 20 min, then extracted with 2x 30ml DCM. The combined DCM extractions was dried over MgSO4 and the solvent removed in a rotovap, leaving the ketone as a yellow oil.

I guess we have a winner..
 
 
 
 
    Aurelius
(Hive Addict)
12-23-02 16:59
No 392052
      another trial on the 2,5-diMeOphenylacetone     

idea and synthesis work by barium

Synthesis of 2,5-dimethoxyphenylacetone
To a three-necked 3000ml rb flask with a stir bar add 166g (1mol) 2,5-dimethoxybenzaldehyde, 15g Aliquat 336, 159g (1.3mol, 139.5ml) methyl 2-chloropropionate, 500ml toluene and 200ml MeOH .With a cold water bath the temperature is brought down to 10-15 deg C. 60g solid NaOH is added in portions of 2-4g over two hours while not allowing the temperature to rise over 15 deg C.

Do not get to brave while adding the hydroxide because the temperature can rise very quick, and you do not want to mop up the haloester from the floor and ceiling – trust me.
When about about 30g NaOH has been added the solution becomes very thick from some precipitate but this will gradually dissolve again. If it becomes too thick to be properly stirred add some more MeOH.

When all NaOH has been added allow the solution to stir for another two hours at room temp.
Then add a pre-made solution of 100g NaOH in 450ml MeOH dropwise during 2 hours while keeping the temperature at 20-30 deg C. Keep it stirring at room temp for another 4 hours and the let it stand over night. The next day the solution will be very thick from precipitated crystals. Add 500ml water and the crystals will dissolve. Separate the two layers and extract the organic layer with another 500ml water. The toluene layer can now be discarged.

To the combined aqueous extracts conc HCl is added until pH 2.5-3 is reached. This will cause a yellow oil to fall out. Allow the acid solution to stir for one hour at room temp, add solid NaCl (enough to saturate the solution) and 500ml toluene and stir for 10 minutes.
Separate the layers and extract the aqueous phase once more with 250ml toluene. Wash the combined toluene phases once with saturated NaHCO3 solution, dry with MgSO4 and remove the toluene by distillation. This leaves 2,5-dimethoxyphenylacetone as a yellow oil.

Yield: 152g (0.78mol) 2,5-dimethoxyphenylacetone

Chloropropionate/bromopropionate/halopropionate can be made using HVZ reaction. Then it is just an easy esterification, you can use the procedure in https://www.rhodium.ws/chemistry/2ch.darzen.html for methyl 2-chloroacetate.

Synthesis of 2,5-DMMA from its acetone as verification for the above synthesis.

2,5-DMMA
To verify the identidy of the 2,5-dimethoxyphenylacetone I decided to N-alkylate it with methylamine.


2,5-dimethoxyphenylacetone 30mmol
methylamine 40mmol
sodium borohydride 50mmol

Methylamine as a 25% aqueous solution was allowed to react with the ketone dissolved in 25ml toluene for 1 hour under violent stirring at room temp. The layers were separated and the toluene layer washed once with 25ml water. The toluene phase was added in one portion to a stabilised aqueous solution of sodium borohydride (the borohydride dissolved in 15ml water containing two drops of 50% aq. NaOH) and the mixture stirred violently for 1.5 hours at room temp. 20% aq HCl added dropwise until pH 2 was reached and the phases separated, the organic phase extractd twice more with 25ml water. The combined aqueous extractions were baified until pH 13 was reached and extracted with 2x30ml toluene. Drying, solvent removal and crystallisation yielded 4.1g (55%) 2,5-dimethoxy-N-methylamphetamine hydrochloride (2,5-DMMA) as bright white crystals.

Crystallisation was performed in EtOAc which is a great solvent for this purpose.
 
 
 
 
    Aurelius
(Hive Addict)
12-23-02 17:11
No 392055
      General Method for Phenylacetones by Darzen Conden     

idea and synthesis method developed by Barium

General Method for Synthesis of Phenylacetones by means of the Darzen Condensation

I´ve made quite a few phenylacetones with this method now, and finally I´ve got a nice overall method.

Method of Synthesis:

1 mol of your favourite benzaldehyde
1.1 mol methyl or ethyl 2-chloropropionate
1.5 mol sodium methoxide, preferebly as the commercially avalible and dirt cheap 30% soln in MeOH.
Toluene, about 200-250 ml/mol benzaldehyde. Make sure the toluene is dry.

Add the toluene, benzaldehyde and haloester to a rb flask equipped with a mag stirbar, thermometer and a addition funnel. With cooling, add the sodium methoxide solution dropwise at a rate which keeps the reaction temperature between 10 to 15°C. When the addition is over allow the mixture to stir at room temp for one hour.
The toluene solution of the glycidic ester is now poured into 1 mol NaOH, as a 5% aqueous solution, and the mixture heated to 70-75°C for 60 minutes to hydrolyze the glycidic ester. The toluene layer is then separated from the aqueous layer. The sodium salt of the glycidic acid can now be isolated by concentration of the aqueous solution to about one third of the original volume and cooling to 10°C, filtration and washing with cold MeOH affords the salt in a quite pure form.

The phenylacetone is generated by heating the previous alkaline solution of the sodium salt of the glycidic acid to 90°C, and acidifying to pH 5.5 with either acetic acid or hydrochloric acid. A vigourous evolution of carbon dioxide starts immediately upon addition of acid. The slightly acidic solution is kept at 90°C for approximately one hour, or about 10 minutes after the carbon dioxide evolution has ceased. The solution is then cooled to room temperature, the pH raised to 8, and the ketone extracted with ether, DCM or toluene.

Yields will vary based on substrate and variations applied to this synthesis.  It cannot be assumed that this procedure is optimized for all benzaldehydes.  However, it gives good yields for nearly all benzaldehydes (at least in those tested)
 
 
 
 
    Aurelius
(Hive Addict)
12-23-02 17:13
No 392057
      Asarone ketone (2,4,5-trimethoxyphenyl-2-propanone     

Found by Rhodium

Asarone ketone, 2,4,5-Trimethoxyphenyl-2-Propanone
A mixture of Glacial Acetic Acid (30ml) and 20-mesh iron (14g, 0.24 mol) in a 250 ml three-necked RBF equipped with a condenser, heating mantle and mechanical stirrer was vigorously stirred and heated at reflux until the mixture became greyish-white (about 30 min). A solution of 2,4,5-trimethoxyphenyl-2-nitropropene (6g, 24 mmol) in glacial acetic acid was added dropwise to this rapidly stirred solution. The reaction mixture was heated under reflux for a total of 3h. The resulting grayish-dark green mixture was vacuum filtered through a bed of celite and then washed with hot acetic acid. The filtrate was then diluted with 100 ml water and extracted with 3x50ml DCM. The combined organic extracts was washed with 5% NaHCO3 and water, dried over MgSO4 and evaporated to give a brown oil. Sublimation at 115-120°C (0.5 mmHg) gave 4g (75%) of the ketone, mp 44-46°C.

Reference: JMC 23, 1318-1323 (1980)
 
 
 
 
    Aurelius
(Hive Addict)
12-23-02 17:16
No 392058
      P2P by Wittig reaction     

Found by Cyrax

The Wittig reaction: phenyl-2-propanone synthesis
The ancient Greeks knew it already: pride comes before a fall.  However, I hope that the gods will forgive me, because I am particulary proud to present you this ingenious synthetic pathway:
*  triphenyl phosphine reacts with alpha-chloroethyl methyl ether to give a slightly hygroscopic phosphonium salt.  However, you can store this salt indefinitely in a sealed container. Yield: 88 %
* after proton abstraction, the phosphorane reacts with benzaldehyde to provide an enol ether (Yield: 88 %) , which can be subjected to hydrolysis (Yield: quantitative).


Procedure:

1) The phosphonium salt was prepared by dissolving triphenyl phosphine (76.4 g, 0.29 mole) and alpha-chloroethyl methyl ether (28.4 g, 0.30 mole) in 125 mL of benzene.  After standing 40 hours the residue was filtered and washed with ether giving a white solid (111 g, 88 % yield) consisting of the crude phosphonium salt containing one molar equivalent of benzene of crystallization.  This crude phosphonium salt is pure enough for the next reaction.  Attempts to recrystallize this salt resulted in partial of complete conversion to a crystalline dihydrate.

2) The phosphonium salt (33 mmoles) was suspended in 40 mL of glyme at - 40 °C under nitrogen.  Potassium t-butoxide (33 mmoles) in 15 mL of glyme was added with stirring over five minutes.  After this point a solution of benzaldehyde (30 mmoles) in 5 mL of glyme was added over five minutes.  The mixture was allowed to warm to room temperature over one hour and was then directly distilled. (Yield: 88 % enol ether)

3) Methyl ketones (phenyl-2-propanone in our case) were conveniently prepared from the enol ethers by mixing with one equivalent of water containing 0.5 % HCl and enough methanol to effect solution.  The solution was heated to boiling and the solvent was then evaporated to give relatively pure samples of ketones in practically quantitative yields.


This reaction is general for the preparation of methyl ketones.  However it seems that the reaction is at its best for the synthesis of P2P (88 %, step 2), since the yield for methyl cyclohexyl ketone is 45 % (step 2), and even worse for 3-methyl-2-nonanone (15 %, step 2).

ref.: D. Robert Coulsen, Tetrahedron Letters (1964) p 3323- 3326
 
 
 
 
    Aurelius
(Hive Addict)
12-23-02 17:40
No 392066
      Nitropropene to P2P     

idea for variation and work done by Barium
originally found and placed in TSII by Rhodium

Synthesis of 1-(2-Flurophenyl)-2-Propanone by Tin (II) with a Nitropropene

Procedure:
18,1g (100mmol) 1-(2-fluorophenyl)-2-nitropropene was added dry in portions to 49,5g (220mmol) SnCl2x2H2O suspended in 75ml EtOAc while the reaction temperature was kept between 20-40 deg C by a cold water bath. When all nitropropene was added and the color had changed to white (5 min) the EtOAc solution was transferred to a rb flask containing 250ml water and 50 ml hydrochloric acid. The EtOAc was removed by distillation under diminished pressure and the aqueous suspension of oxime and tin salts was allowed to stir at 80 deg C for 1 hour. The water phase was now steam distilled to remove the ketone. When no more oily drops came over the distillate was extracted with DCM. The DCM extracts was dried over MgSO4 and the DCM  removed by distillation leaving a quite pure ketone as a colorless oil.

Yield: 13,5g (89%) 1-(2-fluorophenyl)-2-propanone
Purity: 98% (HPLC)
bp 47 deg C @ 0,05mmHg, d 1,077g/ml according to Aldrich. Clear oily liquid with a aromatic smell, quite nice


Tin(II)chloride is quite non-toxic and generally very nice

Logic behind Idea
This is a variation of the Varma and Kabalka method, but they stopped at the oxime stage and said that it was just to extract the oxime with ether. Well the extraction of the oxime from the tin crap/sludge is a NIGHTMARE. So I thought way not render the whole thing acidic and hydrolyse the oxime to the ketone. At the same time the problem with the tin is no more, since Sn(II)chloride and Sn(IV)chloride will stay in the water phase.

Other Examples:

1-phenyl-2-nitropropanone
40,75g (250mmol) 1-phenyl-2-nitropropene
115g (512mmol, 2,05 eq.) SnCl2x2H2O
150ml EtOAc
140ml conc HCl
500mml water

yield; 30,82g (230mmol, 92%) 1-phenyl-2-propanone

1-(3,4-methylenedioxyphenyl)-2-propanone
51,75g (250mmol) 1-(3,4-methylenedioxyphenyl)-2-nitropropene
115g (512mmol, 2,05 eq.) SnCl2x2H2O
150ml EtOAC
140ml conc HCl
500ml water

yield; 41,83g (235mmol, 94%) 1-(3,4-methylenedioxyphenyl)-2-propanone

Varma and Kabalka said that SnCl2/EtOAc doesn´t work with nitroethenes but does so with nitropropenes. I belive that the aldoxime doesn´t survive the enviroment but is hydrolysed to the acetaldehyde, which readily polymerises.

Antoncho's note
in yellium: "Re: PLEASE, anyone..." (Novel Discourse) the aldoximes were made under neutral conditions, while w/sodium stannite only Ph-nitropropenes were successfully reduced.
 
 
 
 
    Aurelius
(Hive Addict)
12-23-02 17:43
No 392067
      P2P from Tin (II) and nitropropene     

synthesis work done by Barium

Synthesis of P2P from phenylnitropropene and Tin (II)

This is a very general procedure and it has so far worked with every single phenylnitropropene I´ve tried. The yields has been in the range of 85-92%. The lower yields are very likely a result of my impatience. Longer time for hydrolysis should have given higher yields.


100mmol of the phenylnitropropene
220mmol Tin(II)chloride dihydrate
200mmol HCl (as a calculated amount conc. aq HCl)
50ml toluene
200ml water
NaCl (solid)

Procedure:

Dissolve the tin(II)chloride in water mixed with the hydrochloric acid. Add the substrate to a rb flask and rinse it all down with the toluene. Now add the aqueous solution to the rb flask in one portion. With good stirring bring the mixture to a gentle reflux for 2 hours. Chill the flask to room temp and saturate the aqueous phase with NaCl, separate the two layers and transfer the top organic layer to a 1L flask containing 500ml water. Steam distill until no more oily drops comes over. This means between 500ml to 1,5L distillate is to be collected depending on the volatility of the ketone. When the steam distillation is over the distillate consits of two layers, one top layer of touene and the major part of the ketone. Separate the layers in a separatory funnel and extract the aqueous phase with 2x100ml toluene. Combine the toluene extracts, dry them with MgSO4 and remove the toluene in a rotovap. This leaves the ketone as a yellow oil ready for whatever you might have in mind for it.

The tin(II)chloride can be regenerated by electrolysis of the formed tin(IV)chloride.

When the hydrolysis (reflux) is over there is actually no need to separate the layers. Just stem distill it as it is. Toulene, ketone and water will pass over, not the tin compounds. One step less
 
 
 
 
    Aurelius
(Hive Addict)
12-23-02 17:47
No 392069
      1-(2,4,5-trimethoxyphenyl)-2-propanone (P2P)     

Synthesis work by Barium

Synthesis of 1-(2,4,5-trimethoxyphenyl)-2-propanone from its nitropropene by Tin (II)


Procedure:
5g (19,8mmol) 1-(2,4,5-trimethoxyphenyl)-2-nitropropene
12,3g (55mmol) tin(II)chloride dihydrate
15ml conc. hydrochloric acid
20ml toluene
15ml water
100ml EtOAc


Tin(II)chloride was dissolved in 15ml water and 15ml conc hydrochloric acid and the resulting solution was added to a 200ml rb flask containing 20ml toluene and 5g substrate. The mixture was heated to 80 deg C on a water bath with good stirring for 1,5 hours. During this time the color changed from a deep yellow to dark orange and then to light yellow.
After 1,5 hours 200ml water was added and the toluene was removed by steam distillation. The now bright yellow aqueous phase containing dark yellow oily droplets was saturated with NaCl and extracted with 2x50ml EtOAc which removed all color. The ethyl acetate was washed once with 50ml saturated NaHCO3 solution, once with 50ml water, once with brine and then dried with MgSO4. The solvent was removed in a rotovap leaving 3,7g of a yellow oil which slowly partially solidified to something reminding of honey.

Yield; 3,7g (83,8%) 1-(2,4,5-trimethoxyphenyl)-2-propanone


Rhodium's Questions:

The nitropropene from asarone stable in hydrochloric acid? Could you please recrystallize your product and verify the melting point of the supposed ketone? Asarylacetone should melt at:

Physical Data:
mp(°C) Source
-------------------------------------------
47-48  Gazz.Chim.Ital.; 36 I; 1906; 283
47-48  J.Chem.Soc.; 1937; 1338,1340
46-47  Helv.Chim.Acta; 47; 1964; 1996-2017
44-46  J.Med.Chem.; 23; 12; 1980; 1318-1323
 
 
 
 
    Aurelius
(Hive Addict)
12-23-02 17:51
No 392070
      P2P from nitroalkenes using Raney Nickel and hypop     

found by Barium

Nitroolefines are converted to the corresponding saturated ketones or aldehydes in high yields by treatment with raney nickel and sodium hypophosphite in aqueous ethanol at pH 5

General experimental procedure:

a suspension of raney nickel (2,5ml) and an aquoeous solution of sodium hypophosphite (14g in 60ml water) were added (in serveral portions and under stirring) to a solution of the nitroolefin (14mmol) in ethanol-aqueous acetate buffer, pH 5 (2:1 ca 400ml). After 2 hours at 40-50 deg C the catalyst was filtered off, water added and the solution extracted with ether. Evaporation of the solvent yielded the carbonyl compound that could be purified by either distillation or crystallisation.

It must be pointed out that under these conditions nitroparaffins are reduced to amines whilst oximes gives the corresponding carbonyl compounds in almost quantitative yields. Ester groups, C=C bonds, nitro and halogen substituents on aromatic nuclei are not affected by this reducing system.

        1-Phenyl-2-nitropropene gives P-2-P in 88% yield
1-(p-MeO-phenyl)-2-nitropropene gives p-MeOP-2-P in 92% yield
1-(p-Br-phenyl)-2-nitropropene gives p-BrP-2-P in 77% yield
1-(o-OH-phenyl)-2-nitropropene gives o-OHP-2-P in 70% yield
1-(p-OH-phenyl)-2-nitropropene gives p-OHP-2-P in 56% yield
1-(p-MeO-phenyl)-2-nitroethene gives p-MeO-phenylacetaldehyde in 53% yield

I have not had the time to try this yet. I found this article yesterday....

If it works it should make 2C-X and DOX within reach by condensation of the benzaldehyde (with everything in place) with nitromethane/ethane..reduction/hydrolysis to the carbonyl and reductive amination to the amine with cyanoborohydride or triacetoxyborohydride

Tet. Lett. vol 24, No 4, page 417-418, 1983
 
 
 
 
    Aurelius
(Hive Addict)
12-23-02 17:58
No 392072
      P2P from Chlorobenzene     

found by 3base

Cl-benzene > allylbenzene > P2Pol > P2P

3-(3-Chloro-2-methylphenyl)propene (17)
sub chlorobenzene here

To a solution of the mono-Grignard reagent prepared[15] from
2,6-dichloro-toluene (48.3g, 0.30mol) in dry THF cooled in
an ice bath was added 1-bromo-3-propene (36.3g, 0.30mol).
The reaction mixture was allowed to warm to room temperature,
stirred for 15h, cooled in an ice bath, and hydrolyzed with
H2SO4/H2O (1:1) until all solids dissolved. The mixture was
extracted with ether, washed, dried, and evaporated. The
product 17 was collected as a colorless liquid at 212-214°C
upon distillation at 760 mmHg: 39.5g (79%)



1-(3-Chloro-2-methylphenyl)propan-2-ol (18)
sub P2Pol here

Compound(17) (5.00g, 30mmol) was added to a solution of mercuric
acetate in water (30mL) and THF (30mL). The reaction mixture
was stirred at room temperature for 30min. Aqueous NaOH (3.0M,
30 mL) was added to the mixture followed by a solution of 0.5M
sodium borohydride in 3.0M NaOH (30mL). After the mercury had
settled, the reaction mixture was extracted with dichloromethane,
washed, dried, and evapo-rated to give a quantitative yield of
the alcohol 18 isolated as a colorless oil: 5.51g (100%);


1-(3-Chloro-2-methylphenyl)propanone (19)
P2P

Jones'reagent[17] (4.1mL) was added dropwise to a solution of the
alcohol(18) (2.00g, 10.8mmol) in acetone (50mL) cooled in an ice
bath. The reaction mixture was stirred for 2min at room tempera-
ture and diluted with water (50mL). The mixture was then extrac-
ted with ether. The organic layer was washed with water, dried,
and evaporated. The crude product was chromatographed on silica
gel using CH2Cl2:hexane 1:1 as eluent to give the ketone(19)
isolated as a pale yellow liquid: 1.20g (60%)


[15] j org chem, 1984 49: 2534; r h mitchell, y -h lai
[17] j chem soc, 1953: 2548; a bowers, t g halsall,
     e r h jones, a j lemin

j org chem, 1996 61(3): 935-940
"Synthesis and Diatropicity of trans-2',5',10b,10c-
Tetramethylfurano[3,4-e]-10b,10c-dihydropyrene. A Valence
Isomerization To Form a Novel Isoannulenofuran at the Expense
of Two Benzene and One Furan Rings"; Yee-Hing Lai, Pu Chen

 
 
 
 
    Aurelius
(Hive Addict)
12-23-02 18:07
No 392074
      P2P from vicinal dihalides - a japanese patent     

dug up and translated "by" PolytheneSam
originally on Rhodium's site

JA44-10776 (MDP2P from dichloroisosafrole)
In reference to Post No 287795

Here's some details on Japanese patent 44-10776 which were read by a Japanese speaking non-chemist to a non-Japanese speaking chemist. 

Note:  The chemical words are written in Katakana (a Japanese alphabet).

In column 1 it discusses making phenylacetone derivatives from phenyl-dihalopropane derivatives and R1 and R2 being either hydroxy, alkoxy or methylenedioxy groups.  In column 2, it looks like it discusses prior art references and also says something about blood pressure.  Lower down column 2 it discusses making the dihalo derivative, but doesn't give details.  It sounds like the dihalo derivatives are made from phenylpropene derivatives.   The translation of examples 1 and 2 are shown below.  BTW, two words in the examples sound a lot like kalium and natrium (potassium and sodium) which might have been borrowed from German, which seems interesting.  I think those words are also the Latin names (or similar) for the elements.  I looked at a Japanese chemical journal before and found both Japanese and German articles in it.

Update:  a Japanese translator told me that a lot of Japanese chemical words were borrowed from German, typically old ones, but some come from English.





Method of making phenylacetone derivatives

Example 1

75 g. of 15% potassium hydroxide is added to 23.3 g. 1-(3,4-methylenedioxyphenyl)-1,2-dichloropropane and agitated 10 hours, cooled to room temperature, extracted twice with 200 ml benzin (benzene or benzine?), dried and evaporated (with heat) to get 15.2 g. 3,4-methylenedioxyphenylacetone(yield 85.4 %).  The boiling point is 149-151º C/10mm Hg.

Example 2

80 g. 20 % sodium hydroxide is added to 33.8 g. 1-(3,4-dimethoxyphenyl)-1,2-dibromopropane (melting point 90-93º C.) and boiled for 12 hours.  After steam distillation and extraction with benzin (benzene or benzine?) 18.4 g. of a light yellow color oil is obtained.  It is pure 3,4-dimethoxyphenylacetone (yield 95.0 %).  The boiling point is 133-137º C/10mm Hg.

----------------


[b]"Yuki et al, Japanese Patent 69:10,776
        Chemical Abstracts 71, 30220e (1969) "[b]

found on https://www.rhodium.ws/chemistry/tcboe/chapter5.htm ?
 
 
 
 
    Aurelius
(Hive Addict)
12-23-02 18:17
No 392075
      possible P2P synth     

idea by PrimoPyro

Bodroux-Chichibabin aldehyde synthesis
A trialkoxymethane (alkyl formate ester dialkyl acetal) reacts with an alkyl or aryl organomagnesium halide to form the intermediate organocarbaldehyde dialkyl acetal, which is then hydrolyzed by acid liberating the organocarbaldehyde


So, say trimethoxy-ethane or triethoxy-ethane, and benzyl magnesium chloride, yields phenylacetone.

How do you produce the trialkoxy-alkane? I found this lovely gem in the same place

Pinner reaction

This reaction takes a organic nitrile, and changes the terminal nitrogen atom into three alkoxy groups, while preserving both the nitrile carbon, and the alkyl chain. Use of this reaction with acetonitrile and methanol or ethanol, will provide one with acquisition of trimethoxy- or triethoxy-ethane, the exact compounds needed for the main reaction to make phenylacetone.

The last step in this reaction may be much higher yielding than the previously known grignard with acetonitrile, and may warrant the good use of adding an extra step to prepare the trialkoxy-ethane.

Think of it as using one reactant, the acetonitrile, to form another reactant, to do the same job. Seems pointless, right? Well maybe not. What if the yields are significantly higher than 40%? What if they are 80%? I would then think it worth it to perform the conversion first, since it does not require any exotic reagents to do so.

If, however, the yields are the same or lower, then I'm just glad to think of it and put the option out there.


Letts Nitrile synthesis

(use to make acetonitrile)

Suggestions for Nitrile synthesis (specifically acetonitrile)

Rhodium:
Aldehydes can be reacted with hydroxylamine to the aldoxime, and then dehydrated with phtalic anhydride under heating (either conventional heating or microwave

Uemura:
 acidamide + P2O5 -> nitrile
 oxime + AA -> nitrile
 
 
 
 
    Aurelius
(Hive Addict)
12-23-02 18:25
No 392078
      P2P synth     

method proposed by Twodogs

New method for P2P

The following reactions are
1) An acid catalysed Aldol condensation of Benzaldehyde and Methyl Ethyl Ketone to give Methyl Phenyl Butenone ie C6H5CHO + CH3CH2COCH3 + dryHCL  ---> C6H5CH=C(COCH3)CH3
2) The unsaturated ketone undergoes the Baeyer-Villiger oxidation with peroxy acids to give the enol ester of Phenyl propanone ie C6H5CH=C(COCH3)CH3  + RCO3H   ----> C6H5CH=C(OCOCH3)CH3
3)The enol ester is then saponified with 10% NaOH solution to give Phenyl Propanone in about 35% yield based on the unsaturated ketone.

The Aldol Condensation.

The directions for this are in Organic Reactions
200 gms of Benzaldehyde and 300 gms of Methyl Ethyl Ketone are mixed in a 1 litre beaker and cooled below 5C. HCL gas is bubbled through until 40gms has been added. The mixture goes from a clear solution to a red colour and becomes turbid so that you can't see through it. The mixture is kept over night and becomes a brown colour. It is washed with water and then 10% NaOH solution, the organic layer seperated and distilled. At 240C a yellow oil comes accross and the temperature gradually rises to 260C.
The oil can be crystalized by cooling in the freezer overnight. This in itself does not induce crystalization but if you also put a spoon in the freezer and then dip it in and out of the cool mixture you get some seed crystals that induce crystalization. The mass turns from an orange oil to sulfur coloured crystals MP 38C  180 gms(Methyl Phenyl Butenone)

The Baeyer-Villiger Oxidation

The reaction of the above unsaturated ketone with peracetic acid was first done by Boesken reported in  Rec. Trav. Chim.  55, 786 (1936). There is some discussion of this also in US patent 3980708.  Also see Organic Reactions Vols 9 & 43 I  think.  (Lugh's contribution reference:)The references from the article on the first condensation from Organic Reactions 16 (entirely on aldol condensations) are from Rec Trav Chem 84 17 & 979 (1968). By following the directions in US Patent   5670661 you will get about 35% ketone based on the weight of the unsaturated ketone used. In that patent it is suggested that by recycling a higher percentage can be achieved.

Procedure:

To a 1 litre flask is added 625 ml of Glacial aceitc acid and 143 grms of Sodium Perborate. To this is added 100 grms of the methyl phenyl butenone with stirring and the mixture is heated to 50C. The mixture will heat up so  care has to be taken ie cooling. However if the mix gets too cool it solidifies. Stirring and heating continued for about 6 hours. Cooled poured into 1 litre H2O and extracted with toluene or DCM. The solvent is distilled leaving a yellow oil that has a pleasant smell. This is added to500 mls of 10% NaOH solution (50/50 H20 EtOH) and stirred for 1-2 hours extracted with toluene or DCM and distilled and the fraction boiling between 210-220C collected Phenyl 2 Propanone (About 35 gms)

In the Organic Reactions review of the Baeyer-Villiger there is a reference to the oxidation of alpha Methyl Cinnamaldehyde using H2O2 catalysed by a nitrobenzene selenic acid  or something like that to give the same enol ester as above but in 90% yield.


Foxy2
Here are some references for the first reaction

Preparation of 3-methyl-3-penten-2-one and its analogs.
Pishch. Prom-st. (Moscow)  (1990),   (10),  44-5.
Journal  written in Russian. 
Abstract
MeCOEt condensed with RCHO (R = Me, Et, Pr, Me2CH, MeOC6H4, PhCH:CH, Ph) in the presence of H2SO4 at 60-65° to give 40-79% RCH:CMeCOMe.  Analogous reaction of R1CH2COR2 [R1R2 = (CH2)3, (CH2)10; R1 = H, R2 = Ph, CH2CHMe2; R1 = C5H11, R2 = Me] with MeCHO (from paraldehyde) gave 8.6-38% MeCH:CR1COR2. 


Aldol condensation of butanone with various aldehydes.    
Sasaki, Kazuhiro.    Kobayashi Perfum. Co.,  Ichikawa,  Japan.   
Nippon Kagaku Zasshi  (1968),  89(8),  797-804. 
Journal  written in Japanese.   
Abstract
Condensation of MeCOEt (I) with various aldehydes was investigated to find the effect of conditions on the ratio of RCH:CHCOEt (II) and RCH:CMeAc (III).  Citral and I, in the presence of alkali hydroxide or NaOEt, gave products contg. >86.8% II, whereas MeOH-KOH yielded products contg. III as a main product.  Citronella (IV) and I gave similar results except that Me2C:CHCH2CH2CHMeCH2CH(OMe)CH2COEt was formed using MeOH-KOH and Me2C:CHCH2CH2CH(Me)CH2CH(OH)CHMeAc was obtained with EtONa.  The results indicate that aldol corresponding to II is more readily dehydrated than that corresponding to III.  Condensation of IV with Me2CO in the presence of MeOH-KOH gave a mixt. of Me2C:CHCH2CH2CHMeCH2CH:CHAc and Me2C:CHCH2CH2CHMeCH2CH(OMe)CH2Ac.  Similar reaction of I with Et2CO gave Me2C:-CHCH2CH2CHMeCH2CH:CMeCOEt, b0.55 101-7°.  Condensation of I with BzH, PhCH:CHCHO (V) and furfural (VI) showed that the ratio II-III is also dependent on the type of aldehyde.  III is favored when KOH-MeOH is used with BzH and V, compared with the results with aq. NaOH, but the difference is much greater with BzH.  BzH and VI always favor formation of II.  PhCH:CHCH:CHCOEt, m. 55-6°, was prepd. by a modified Wittig reaction; semicarbazone m. 200-1°; phenylhydrazone m. 89-90°.  4-(2-Furyl)-3-methyl-3-buten-2-one, b10 114-15°, and 5-(2-furyl)-4-penten-3-one, b13 122-4°, were also prepd. by this method.  Orientation of condensation was postulated to be regulated by the steric requirement.  s-Cis and s-trans conformations of some of the unsatd. ketones were detected by ir spectra. 


A study of the reaction of butanone with benzaldehyde and p-nitrobenzaldehyde.    
Jung, Duksang.    Cheju Univ.,  Cheju,  S. Korea.  
Nonmunjip - Cheju Taehak  (1982),  14  27-31.
Journal  written in Korean.   
Abstract
In alk. medium, the reaction of p-O2NC6H4CHO and MeCOEt gave three hydroxy ketones, indicating that both Me and CH2 positions were attacked to a comparable extent.  Dehydration of the intermediate hydroxy ketones is a slow step.  In acid medium, however, the addn. step of the reaction was selective, giving only 1 isomer.  Similarly, in alk. medium, PhCH(OH)CH2COEt (I) and PhCH(OH)CHMeCOMe (II), hydroxy ketone intermediates from the reaction between BzH and MeCOEt, gave PhCH:CHCOEt (III).  Treating I and II with acid gave III and PhCH:CMeCOMe, resp., with no evidence of rearrangement. 


These journal articles detail its synth also.

Double Michael addition reactions of some new 1,5-diaryl-2-alkyl-1,4-pentadien-3-ones: Part II.
Indian J. Chem., Sect. B: Org. Chem. Incl. Med. Chem.  (2001),  40B(8),  667-673.

RuCl3 catalyzes aldol condensations of aldehydes and ketones.   
Tetrahedron  (1998),  54(32),  9475-9480. 

Synthesis of isoquinoline-1,3-dicarboxylic acid.    
Chin. Chem. Lett.  (1999),  10(11),  907-910. 
(in english)

Here is the baseic addition

Kinetics of condensation of benzaldehyde and its derivatives with acetone and methyl ethyl ketone catalyzed by aluminum oxide.   
Collect. Czech. Chem. Commun.  (1980),  45(6),  1812-19.
Journal  written in English.
Abstract
The pseudo-1st-order aldol condensation kinetics of RC6H4CHO (R = H, 4-Me, 3-MeO, 4-MeO, 3-Cl,4-Cl) with excess Me2CO over Al2O3 at 60-160° and the pos. r indicated that the addn. step, to give hydroxy ketone, is rate detg.  r Decreases with increasing temp.; the isokinetic temp. is 449.8K.  At 60-90° the retroaldol reaction of PhCH(OH)CH2Ac is minor and the dehydration to PhCH:CHAc is major; the proportion of dehydration-retroaldol reaction increases with increasing solvent polarity and decreases with increasing temp.  The condensation of PhCHO with MeCOEt at 90-160° gives mostly PhCH:CHCOEt in a reaction catalyzed by the basic sites on Al2O3; the minor product, PhCH:CMeAc, formation is catalyzed by the acidic Al2O3 sites.

This review would probably bee very helpful.

The Baeyer-Villiger oxidation of ketones and aldehydes.
Krow, Grant R.
Org. React. (N. Y.)  (1993),  43  251-798.  
Abstract
A review with >1092 refs. 


Refs for the second one(partial of this article)

Sn-zeolite beta as a heterogeneous chemoselective catalyst for Baeyer–Villiger oxidations
Nature 412, 423 - 425 (2001)

The Baeyer–Villiger oxidation, first reported more than 100 years ago1, has evolved into a versatile reaction widely used2 to convert ketones—readily available building blocks in organic chemistry—into more complex and valuable esters and lactones. Catalytic versions of the Baeyer–Villiger oxidation are particularly attractive for practical applications, because catalytic transformations simplify processing conditions while minimizing reactant use as well as waste production. Further benefits are expected from replacing peracids, the traditionally used oxidant, by cheaper and less polluting hydrogen peroxide3. Dissolved platinum complexes4 and solid acids, such as zeolites5, 6 or sulphonated resins7, efficiently activate ketone oxidation by hydrogen peroxide. But these catalysts lack sufficient selectivity for the desired product if the starting material contains functional groups other than the ketone group; they perform especially poorly in the presence of carbon–carbon double bonds. Here we show that upon incorporation of 1.6 weight per cent tin into its framework, zeolite beta acts as an efficient and stable heterogeneous catalyst for the Baeyer–Villiger oxidation of saturated as well as unsaturated ketones by hydrogen peroxide, with the desired lactones forming more than 98% of the reaction products. We ascribe this high selectivity to direct activation of the ketone group, whereas other catalysts first activate hydrogen peroxide, which can then interact with the ketone group as well as other functional groups.


When trying to avoid the use of peracids for the Baeyer–Villiger reaction, the methodology developed up to now involved catalysts able to activate hydrogen peroxide, H2O2. Amongst homogeneous catalysts, complexes of molybdenum8 and rhenium9 have been shown to activate H2O2, but their turnover numbers (TONs) and selectivities are relatively low (TONs are below 20 for overall reaction times ranging from 12 to 24 hours). Pt complexes achieve TONs of about 50 within 5 hours, but are not chemoselective when other functional groups are present10. Concerning heterogeneous catalysts, supported Pt complexes11 and TS-112 have been used, but have shown only limited activity and selectivity, respectively. Acid zeolites such as H-beta and USY activate hydrogen peroxide for the Baeyer–Villiger oxidation, but show selectivities of less than 60–70% (ref. 5). MeReO3 (ref. 13), TS-13 and Pt complexes14 are excellent epoxidation catalysts in the presence of H2O2 and consequently, epoxidation is favoured over the Baeyer–Villiger oxidation when using unsaturated ketones as starting material.

References
1. Baeyer, A. & Villiger, V. Einwirkung des Caro'schen Reagens auf Ketone. Chem. Ber. 32, 3625-3633 (1899).
2. Renz, M. & Meunier, B. 100 years of Baeyer-Villiger oxidations. Eur. J. Org. Chem. 737-750 (1999).
3. Arends, I. W. C. E., Sheldon, R. A., Wallau, M. & Schuchardt, U. Oxidative transformations of organic compounds mediated by redox molecular sieves. Angew. Chem. Int. Edn Engl. 36, 1145-1163 (1997).
4. Strukul, G. Transition metal catalysis in the Baeyer-Villiger oxidation of ketones. Angew. Chem. Int. Edn Engl. 37, 1198-1209 (1998).
5. Fischer, J. & Hölderich, W. F. Baeyer-Villiger-oxidation of cyclopentanone with aqueous hydrogen peroxide by acid heterogeneous catalysis. Appl. Catal. A 180, 435-443 (1999).
6. Chang, C. D. & Hellring, S. D. Production of lactones and omega-hydroxycarboxylic acids. US Patent No. 4870192 (1996).
7. Hoelderich, W., Fischer, J., Schindler, G. -P. & Arntz, D. Preparation of lactones by Baeyer-Villiger oxidation of cyclic ketones. German Patent DE 19745442 (1999).
8. Jacobson, S. E., Tang, R. & Mares, F. Oxidation of cyclic ketones by hydrogen peroxide catalysed by group 6 metal peroxo complexes. J. Chem. Soc. Chem. Commun. 888-889 (1978).
9. Herrmann, W. A., Fischer, R. W. & Correia, J. D. G. Multiple bonds between main-group elements and transition metals. Part 133. Methyltrioxorhenium as a catalyst of the Baeyer-Villiger oxidation. J. Mol. Catal. 94, 213-223 (1994).
10. Gavagnin, R., Cataldo, M., Pinna, F. & Strukul, G. Diphosphine-palladium and -platinum complexes as catalysts for the Baeyer-Villiger oxidation of ketones: effect of the diphosphine, oxidation of acyclic ketones, and mechanistic studies. Organometallics 17, 661-667 (1998).
11. Palazzi, C., Pinna, F. & Strukul, G. Polymer-anchored platinum complexes as catalysts for the Baeyer-Villiger oxidation of ketones: preparation and catalytic properties. J. Mol. Catal. A 151, 245-252 (2000).
12. Bhaumik, A., Kumar, P. & Kumar, R. Baeyer-Villiger rearrangement catalysed by titanium silicate molecular sieve (TS-1)/H2O2 system. Catal. Lett. 40, 47-50 (1996). 
13. Herrmann, W. A. Essays on organometallic chemistry, VII. Laboratory curiosities of yesterday, catalysts of tomorrow: organometallic oxides. J. Organomet. Chem. 500, 149-150 (1995).
14. Frisone, M. D. T., Pinna, F. & Strukul, G. Baeyer-Villiger oxidation of cyclic ketones with hydrogen peroxide catalyzed by cationic complexes of platinum(II): selectivity properties and mechanistic studies. Organometallics 12, 148-156 (1993).
 
 
 
 
    Aurelius
(Hive Addict)
12-23-02 18:31
No 392080
      P2P     

found by Rhodium

New synthesis of Phenyl-2-propanones
Phenyl-2-propanone [Tet Lett 29(24), 2977-2978 (1988)]


Procedure:

To a stirred solution of 2-nitropropene (0.1 mol, 8.7g) in dry CH2Cl2 (300ml) was added benzene (0.5 mol, 39g) at room temperature. Titanium tetrachloride (0.1 mol, 19g) was then added dropwise into the mixture with stirring at the same temperature. After being stirred for 60 min (or when the starting material completely disappears on TLC), water (150 ml) was added and the resultant heterogenous mixture was stirred at reflux for 2h. The organic phase was separated, the aqueous phase extracted with CH2Cl2, and the pooled organic extracts washed with 1M Na2CO3 solution and dried over MgSO4. Evaporation of the solvent followed by vacuum distillation (bp 100-101°C at 14mmHg) afforded Phenyl-2-propanone (ca 9g, 70% of theory).

2-nitropropene can be bought, or synthesized as outlined below:

2-Nitropropanol [JACS 67, 205 (1945)]

75.1g Nitroethane, 0.3g calcium hydroxide and 80g 40% formaldehyde solution was dissolved in 75ml ethanol with stirring and was allowed to stand for 48h at room temperature. Distillation at 100-105°C/13 mmHg (85-86°C/6 mmHg, 99°C/10 mmHg) gave 48g 2-nitropropanol (46%) and 14.3g of 2-nitro-2-methyl-1,3-propanediol, the latter remained as a crystalline residue in the distillation flask after distillation of the 2-nitropropanol.

2-Nitropropyl acetate [Ind Eng Chem 32, 34 (1940)

105 grams of 2-nitropropanol (1 mol) was placed in a 250ml two-necked RB flask equipped with a reflux condenser and an addition funnel, and 110g of acetic anhydride (1.078 mol) was added dropwise with good stirring. The solution was then refluxed for 30 minutes, cooled and poured into 200ml cold water, the organic phase separated and washed with 100ml water, and then vacuum distilled (bp ca 100°C at 10 mmHg, yield 90% ). The acetate is a clear water-white liquid with a faint, slightly mustardlike odor, almost insoluble in water.

2-Nitropropene [JOC 15, 8 (1950)]

Sodium carbonate (0.25 mol, 14.5g) is added to a solution of 2-nitropropyl acetate (0.5 mol, 73.5g) dissolved in 50ml benzene and refluxed in a RB flask for six hours. The flask is cooled and the product decanted from the solids. The residue (consisting of sodium acetate) is extracted with 50 ml benzene, dissolved in 100ml water, and the solution extracted with 3x25ml benzene. The pooled organic extracts are dried over anhydrous MgSO4, and the mixture fractionally distilled to afford 2-nitropropene (bp 57°C at 100 mmHg).
 
 
 
 
    Aurelius
(Hive Addict)
12-23-02 18:33
No 392082
      electro synth of P2P     

found by WizardX

Yes the reaction is workable, however not as easy as you may think. The P2P synthesis from benzyl chloride + AA is one that gives good results with a skilled operator.

P2P SYNTHESIS via electrosynthesis of ketones.

Ref - Tet.Lett 27,35, (1986) 4175-4176
EP198743A1 Process for the electrosynthesis of ketones.
EP198743B1 Process for the electrosynthesis of ketones.


Procedure:

30 mmoles of Benzyl Chloride (3.79g), an excess of Acetic Anhydride (>3.5g) and 1mmole Tetrabutylammonium Iodide (0.37g) or 1mmole tetrafluoro borate (0.09g) are dissolved in 25ml of DMF.
This is transferred to an Electrolytic Cell with the anode being a rod of Magnesium (Mg) or Duralin (Al) (1cm diameter) and the cathode being a cylindrical grid of Stainless Steel or Nickel. The cell is immersed in an  ice water bath so that the temp stays between 0C and 10C. A constant current intensity (0.4A) is applied until 3 to 4 moles of electrons per mole of Benzyl Chloride (in this case 90-120mmoles) are passed.

The contents of the cell are transferred to a flask and refluxed with 20% H2SO4 for 1 hr in order to destroy the excess of Acetic Anhydride and release the ketone from the enol ester which may be formed.

The mixture is then extracted several times with ether (or other non-polar solvent?), and the extract is vacuum distilled.
 
 
 
 
    Aurelius
(Hive Addict)
12-23-02 18:43
No 392084
      P2P by borane reagents     

found by Obituary

P2P synth using phenylborane

triphenylborane + bromoacetone -(OH-)-> P2P

this seems very reasonable considering that alkylboranes will react with bromo-keto comps to give ketones under basic conditions

how available is triphenylborane?

Lugh
These borane syntheses and the related nitrile syntheses are old and well known to many. Some other related refs are JACS 90: 5280,5936 & 6891 (1968) & 91: 4304, 6852 & 6854

Diazoeacetone can be used as an alternative to bromoacetone. They work very well with the substituted aromatics. While not the cheapest, these routes offer the advantage of being generally applicable, and thus very suited for our purposes
 
 
 
 
    Aurelius
(Hive Addict)
12-23-02 20:44
No 392115
      Ephedrine to P2P     

Patent found by Bwiti
Translated to english by LabTop

P2P KETONE FROM EPHEDRINE(-TABLETS or BULK)


Procedure for the Production of Phenylacetone (P2P).
Patent # :  DE3200232
(addition for patent application P 30 26 698.9-42)


The invention concerns a new arrangement of a procedure for the production of phenylacetone.

Reaction Details:[b]

The subject of the principal patent (patent application P 30 26 698,9-42) is marked as a procedure for the production of Phenylacetone from Ephedrine by sulfuric acid (H2SO4), by the fact that one executes the conversion to P2P with 50-70 % acid at 150 to 155*C and in the presence of 0,02-0.5 % zinkchloride as a catalyst and that one continuously distillates the formed Phenylacetone out by means of steam distillation, directly from the beginning of the reaction.
It was now found that one can execute the conversion also in the presence of 0,05 to 0.3 % of other metalchlorides in place of 0,02 to 0.5 % zinkchlorid.

As other metalchlorides are suitable Iron(III)chloride and in particular aluminumchloride, borontrifluoride and Titan(III)chloride, which are used as Lewis acids in organic chemistry.

The concentration of the sulfuric acid is selected in such a way, that it
amounts to a content in the reaction mixture of 50-70 weight %.
The reaction runs particularly well with a weight/weight(w/w) sulfuric acid content from approximately 60 %.

For the continuous discharge of the Phenylacetone from the reaction mixture, water vapour is initiated into the mixture, which removes the developing Phenylacetone immediately from the mixture.

The received distillate is extracted with toluene and the latter is removed by distillation. Thus one obtains Phenylacetone, whith a purety of
over 99,5 %.

For a source of Ephedrine comes Ephedrine, Pseudoephedrine, Norephedrine and Norpseudoephedrine as well as Bis-(1phenyl-2-methylaminopropyl1)-ether in consideration, whereby the reactionmechanism for such Ephedrines is particularly important, no direct results for so far for (-)-pseudoephedrin and (-)-norpseudoephedrin. The Ephedrine preferably is used in a weight/weight ratio of approximately 1:1 to 1:10, preferably 1:2 to 1:5 compared to the acid.

The yields, which can be obtained with the new procedure, are within the  80 % range. The procedure runs at a relatively high rate/min.

A further advantage of the procedure consists of the fact that it can be executed continuously . Thus one can let a hot aqueous solution of the Ephedrine of choice flow into the hot acid, whereby the  Phenylacetone/Water azeotrope continuously gets distillated out. Here it is only necessary, to remove the developing ammonium salt, e.g. methylammoniumhydrogensulfat occasionally.

The use of the mentioned metal halides in place of ZnCl2 offers advantages regarding environmental protection, because these are more harmless and do not disturb the biological reduction of the waste water.

Example 1:

1025 g 75 % sulfuric acid are mixed with 2 g AlCl3. In there one dissolves 426 g (= 2.58 mol) Ephedrine or Pseudo-ephedrine derivate from 50-100*C. Subsequently, the mixture is heated up to 145-150*C. At 125*C steam in moderate current is introduced in the fluid for better mixing. At 145*C one increases the steam introduction and distills in a period of 2 1/2 to 3 hours the Phenylacetone/Water azeotrope over. From the distillate one isolates by toluene extraction the raw Phenylacetone, which is free from Propiophenon. After distillation with a short Vigreux reflux column attached also, one receives Phenylacetone, yield 270 g (78%)with a purity upto 99.8 %.
One gets similar yields from Nor-ephedrine or Nor-pseudo-ephedrine for
Phenylacetone in a yield of approx. 80 %.


Example 2

In 1000 g 79 % sulfuric acid, an amount of 100 g (-)-Pseudoephedrin-derivate gets dissolved, whereby the H2SO4-concentration is adjusted to 65 to 66 weight%. This solution is poured into one of the necks of a three-neck roundbottomflask, mixed with 4 g AlCl3 and heated up to 125-130*C.
By injecting steam, one increases the temperature to 145-150*C and keeps it going in a steady rate from now on with a flow-rate of approximately 1 to 5 g/min.
The 70-90*C warm solution of the (-)-pseudoephedrin(derivate) mixed with
79 % sulfuric acid solution(weight ratio 1:1) is added by means of a steam-heated dropping funnel. The Phenylacetone turns thereby into an
azeotrope and is similar to example 1 regenerated.

In both cases, after 4 hrs, the reaction is interrupted for 5 minutes and during that 5 minutes there is so much sulfuric acid removed that the original volume is re-created. Then the reaction is continued. The removed sulfuric acid can be re-used after one removes the methylammoniumhydrogensulfat developed during the conversion. The yield of Phenylacetone amounts to 76 %.

Example 3

1350 g 79 % sulfuric acid is added through a reactionflask neck and mixed with 590 g 95 % (+)-Ephedrine.
10 ml 18 % TiCl3 is added subsequently, the solution mixed and the mixture heated to 125*C, and then heated to 145-155*C with steam, the distillate collected, from which by toluene extraction 358 g Phenylacetone were isolated. From this, 335 g = 73.6 % yield of Phenylacetone, with a purity of 99.5 % was collected by fractionated distillation over a Vigreux column.

Example 4

In 1350 g 79 % sulfuric acid is dissolved under agitating 561 g d-l-Ephedrin (99-100 %), mixed with 15 g BF3 / glacial acetic acid solution (10 %) and heated to 125*C.
One injects steam and increases the temperature thereby to 145-155*C whereby developed Phenylacetone is collected. After approx. 6 L steam distillate is collected, the conversion is terminated. One isolates 386.4 g Phenylacetone from the distillate by toluene extraction and distillation. From this, 367 g (80.6 %) pure Phenylacetone is collected .

Example 5

To 400 kg 60 % sulfuric acid added in a distillation apparatus, 500 kg of 80 % sulfuric acid are being gear-pumped. Added to this mixture is 400 kg (-)-pseudoephedrine (60 %) under mixing, and 1.5 L aluminum chloride solution (30 %) introduced. One heats the reaction mixture to 125-130*C and introduces then steam, while increasing the temperature to 145-150*C ,
whereby a mixture of steam/Phenylacetone distills over. After approx. 2000 1 steamdistillate is collected, one terminates distillation. From the distillate the lower heavy oil phase, consisting of Phenylacetone, is separated and the aqueous phase is extracted with approx. 400 1 toluene. The separated Phenylacetone and the toluene extracts are combined and concentrated by distillation. The remaining arrears are afterwards fractionated distilled. One receives pure Phenylacetone to 155 kg (80 %).


[b]Note by Terbium:


But according to the patent you would not seem to need anhydrous materials. Steam is injected into the pot as the reaction is proceding in order to steam distill off the P2P as it is formed. The patent even talks about doing this as a continuous process where ephedrine is also continously added.
It seems to me that hydrated aluminum chloride should work just fine. Also, zinc chloride in most any form should work.   

Note by Jim:
 
My suggestion is:  Don't steam distill, it will complicate things.  Take the sulfuric acid and ZnCl2, put into flask.  Heat and add the crude ephedrine pills.  The ephedrine turns into P2P, the fillers are chewed up.  Extract with nonpolar like toluene after reaction has run its course and cooled down.  Use P2P in any fashion you like to get racemic amphetamines. 

Note by LabTop:

For those of you who have access to bulk amounts of pure ephedrine, like in asia, from MaHuang, this is the preferred way of making P2P in continuous big scale operation there.
Steam distillation is then preferred.
After that, you convert the P2P with the OnePot Meth ICE method to racemic d,l-methamphetamine, and with the help of d-tartaric acid, convert that to d-methamphetamine, and l-methamphetamine.
Or use hydrogenation techniques. LT/
 
 
 
 
 
    Aurelius
(Hive Addict)
12-23-02 21:10
No 392126
      PhBr and acetone enol reaction to P2P     

Reposted by Dormouse
Refs found by Rev. Drone originally

Enolate chemistry is water! P2P from acetone and phenyl halide.

There is a great deal of material on the general topic of phenylacetones from halobenzenes. I looked at the papaer, and still I can't see what you're talking about. Please, tell me what paragraph, what line, because I just don't see it in the article you cite.
I have a large list of ref's for this, but my practical experience with this is limited.

Here are some ref's culminated from Beilstein on this.

1800928; Journal; Mousseron; Winternitz; BSCFAS; Bull.Soc.Chim.Fr.; 1946; 604, 606;
1837235; Journal; Leake; Levine; JACSAT; J.Amer.Chem.Soc.; 81; 1959; 1169, 1171;
121365; Journal; Allard,M.; Levisalles,J.; BSCFAS; Bull.Soc.Chim.Fr.; FR; 1972; 1926-1931;
188314; Journal; Rossi,R.A.; Bunnett,J.F.; JOCEAH; J.Org.Chem.; EN; 38; 1973; 3020-3025;
20359; Journal; Bunnett,J.F.; Sundberg,J.E.; CPBTAL; Chem.Pharm.Bull.; EN; 23; 1975; 2620-2628;
191322; Journal; Bunnett,J.F.; Sundberg,J.E.; JOCEAH; J.Org.Chem.; EN; 41; 1976; 1702-1706;
192362; Journal; Scamehorn,R.G.; Bunnet,J.F.; JOCEAH; J.Org.Chem.; EN; 42; 8; 1977; 1449-1457;
5571095; Journal; Bard, Raymond R.; Bunnett, Joseph F.; JOCEAH; J.Org.Chem.; EN; 45; 8; 1980; 1546-1547;
5625248; Journal; Alonso, Ruben A.; Rossi, Roberto A.; JOCEAH; J.Org.Chem.; EN; 45; 23; 1980; 4760-4763;
5831694; Journal; Beugelmans, Rene; Ginsburg, Helene; JCCCAT; J.Chem.Soc.Chem.Commun.; EN; 11; 1980; 508-509;
5672337; Journal; Sugai, Saburo; Ikawa, Hiroshi; Okazaki, Tokuji; Akaboshi, Sanya; Ikegami, Shiro; CMLTAG; Chem.Lett.; EN; 1982; 597-600;
5692294; Journal; Kuwajima, Isao; Urabe, Hirokazu; JACSAT; J.Amer.Chem.Soc.; EN; 104; 24; 1982; 6831-6833;
5692587; Journal; Beugelmans, Rene; Ginsburg, Helene; Bois-Choussy, Michele; JCPRB4; J.Chem.Soc.Perkin Trans.1; EN; 1982; 1149-1152;
5747425; Journal; Hamana, Masatomo; Iwasaki, Genji; Saeki, Seitaro; HTCYAM; Heterocycles; EN; 17; 1982; 177-181;
5747627; Journal; Iwasaki, Genji; Hamana, Masatomo; Saeki, Seitaro; HTCYAM; Heterocycles; EN; 19; 1; 1982; 162;
5578586; Journal; Fox, Marye Anne; Younathan, Janet; Fryxell, Glen E.; JOCEAH; J.Org.Chem.; EN; 48; 18; 1983; 3109-3112;
5866684; Journal; Kosugi, Masanori; Hagiwara, Isao; Sumiya, Takashi; Migita, Toshihiko; JCCCAT; J.Chem.Soc.Chem.Commun.; EN; 7; 1983; 344-345;
5575366; Journal; Galli, Carlo; Bunnett, J. F.; JOCEAH; J.Org.Chem.; EN; 49; 16; 1984; 3041-3042;
5577310; Journal; Scamehorn, Richard G.; Hardacre, Jerry M.; Lukanich, Jeanne M.; Sharpe, Lee R.; JOCEAH; J.Org.Chem.; EN; 49; 25; 1984; 4881-4883;
5743762; Journal; Kosugi, Masanori; Hagiwara, Isao; Sumiya, Takao; Migita, Toshihiko; BCSJA8; Bull.Chem.Soc.Jpn.; EN; 57; 1; 1984; 241-246;
5617267; Journal; Bunnett, Joseph F.; Galli, Carlo; JCPRB4; J.Chem.Soc.Perkin Trans.1; EN; 1985; 2515-2520;
5758633; Journal; Amatore, Christian; Oturan, Mehmet A.; Pinson, Jean; Saveant, Jean-Michel; Thiebault, Andre; JACSAT; J.Amer.Chem.Soc.; EN; 107; 1985; 3451-3459;
5610537; Journal; Galli, Carlo; TETRAB; Tetrahedron; EN; 44; 16; 1988; 5205-5208;
5868096; Journal; Galli, Carlo; GCITA9; Gazz.Chim.Ital.; EN; 118; 5; 1988; 365-368;
5569229; Journal; Fukazawa, Yoshimasa; Usui, Shuji; Kurata, Yoshiyuki; Takeda, Yoshiya; Saito, Nahoko; JOCEAH; J.Org.Chem.; EN; 54; 12; 1989; 2982-2985;
5813817; Journal; Galli, Carlo; Gentili, Patrizia; JCPKBH; J.Chem.Soc.Perkin Trans.2; EN; 6; 1993; 1135-1140;
5996665; Journal; Galli, Carlo; Gentili, Patrizia; Guarnieri, Alessandra; GCITA9; Gazz.Chim.Ital.; EN; 125; 9; 1995; 409-412;
6089128; Journal; Hamann, Blake C.; Hartwjg, John F.; JACSAT; J.Amer.Chem.Soc.; EN; 119; 50; 1997; 12382-12383;
6091280; Journal; Palucki, Michael; Buchwald, Stephen L.; JACSAT; J.Amer.Chem.Soc.; EN; 119; 45; 1997; 11108-11109;

 DOES ANYBODY HAVE ADDITIONAL INFORMATION ON THIS REACTION?  IS THERE A STANDARDIZED PROCEDURE YET?  IF YOU HAVE INFORMATION ON THIS, PLEASE POST A SEPARATE RESPONSE THREAD
 
 
 
 
    PolytheneSam
(Master Searcher)
12-23-02 23:57
No 392172
User Picture 
      more     

I always liked this one since it uses a plastic monomer.
Post 43910 (CheshireHouse: "alpha-methyl styrene", Serious Chemistry)

Phenylacetic acid --> phenylacetone is classic.
Post 18563 (Mastermind: "P2P", Methods Discourse)

This one is strange.  There seems to be one too many N's on the right side of the equation in column 3 (see claim 1).  It appears that you get o-hydroxyphenylacetone imine from bromobenzene and acetone oxime, for example.
Patent US3547997


http://www.geocities.com/dritte123/PSPF.html
The hardest thing to explain is the obvious
 
 
 
 
    Rhodium
(Chief Bee)
12-26-02 00:14
No 392835
User Picture 
      alpha-methylstyrene     

The full alpha-methylstyrene procedure: https://www.rhodium.ws/chemistry/2-phenylpropanal.html

Sam: It's just a typo in column 3, it says "NCl" instead of "HCl".
 
 
 
 
    Aurelius
(Hive Addict)
12-26-02 16:56
No 393003
      More on Enol P2P
(Rated as: excellent)
    

Rhodium sent the following to Aurelius as a follow-up on Post 392126 (Aurelius: "PhBr and acetone enol reaction to P2P", Methods Discourse)

Rhodium
Acetone + Benzene -> P2P
Check out https://www.rhodium.ws/pdf/archive/alpha-arylation.html

It is a review article about alpha-arylation of ketones, in other words methods which can be used to attach acetone to a benzene ring, forming phenylacetone (P2P). I don't have the time to chase all those references, but somebody else may be interested in doing that some rainy day?

Slappy
That is why I present...

Synthesis of substituted Phenylacetone derivatives via Pd-catalysed alpha-Arylation of Acetoacetate esters.

The efficent large scale synthesis of Phenylacetone and it's derivatives can be accomplished from economical precursors via a Pd-catalysed coupling reaction, and subsequent decarboxylation. First, Methyl (or Ethyl) Acetoacetate is alpha-Arylated with an Aryl Halide (Ph-Br, Ph-I, Ph-OTf), to form 1-Carbomethoxy-Phenylacetone. This ester is then decarboxylated by boiling in dilute HCl, yielding P2P. The attractivness of this route is that it starts from cheap, commodity chemicals (Bromobenzene, Ethyl Acetoacetate, and tribasic Potassium Phosphate are less than $30/kg), low catalyst loading (1-2 mol% Pd), and high efficiancy (yield 80-90%).

The mechanism involves oxidative addition of Ar-X to the Pd center to give Ln(Ar)Pd-X (where L=1 or 2),which then coordinates to the enolate (formed by the beta-Ketoester and base, like K3PO4), and eliminates a mole of KX, giving Ln(Ar)Pd-Enolate, which couples the coordinated Aryl moiety, and reductivly eliminates.

Aryl Halide (1 eq)  = R-Ph-Br, R-Ph-I, R-Ph-OTf
Alkyl Acetoacetate (1.2 eq)
Base (2 eq) = K3PO4
Pd pre-catalyst (1 mol%) = Pd(OAc)2, Pd2(dba)3
Phosphine ligand (2.2 mol%) = PPh3, PtBu3, PCy3

Sutible solvents:
THF, Toluene, Dioxane

Reaction conditions:
10-48h at 70-100°C


Ref:

Buchwald et al., "Highly Active and Selective Catalysts for the Formation of a-Aryl Ketones", J. Am. Chem. Soc. 2000, 122, 1360-1370
Hartwig et al., "Transition metal-catalysed process for preparing alpha-arylated carbonyl containing compounds", Patent US6057456
 
 
 
 
    Barium
(Hive Bee)
12-28-02 11:35
No 393459
      OMG!!     

The last method seems to be a beauty tongue
 
 
 
 
    Aurelius
(Hive Addict)
12-28-02 23:58
No 393610
      Barium     

Barium, for shame!!! you haven't seen these?  the bee who is such an individual in the field of P2P synthesis?!

there's more too, not specifically on that subject just yet, but aurelius has a couple more to add before the compilation is up-to-date with aurelius' notes
 
 
 
 
    Megatherium
(Stranger)
12-29-02 01:16
No 393653
      A beauty, yes ... but it 's a shame they use...     

A beauty, yes ... but it 's a shame they use such an exotic phosphine ligand.  I don't know the exact name, so I 'll describe it: it 's a biphenyl which is ortho substituted on both rings, in one ring the ortho substituent is methyl and the other ring has a -P(t-Bu)2 group in the ortho position.

Here 's the procedure:
An oven-dried, resealably Schlenk tube containing a stirbar was capped with a rubber septum, evacuated, and cooled under argon.  The tube was then charged with Pd (OAc)2 (2.3 mg, 0.01 mmol), ligand (6.9 mg, 0.022 mmol), and K3PO4 (490 mg, 2.3 mmol).  The septum was replaced, and the tube was again evacuated and filled with argon.  THF (2 ml), diethyl malonate (1.2 mmol), and 4-bromo-tert-butylbenzene (1 mmol) were injected, and under a flow of argon, the septum was replaced by a Teflon screw cap.  The tube was sealed and heated at 70 °C for 10 hours.  The mixture was then diluted with ethyl acetate and filtered.  The filtrate was concentrated and chromatographed to give 266 mg (91 %) of diethyl alpha-(4-tert-butylphenyl)malonate. as an oil.

Read aceto acetate if you see diethyl malonate smile.
 
 
 
 
    Lego
(Newbee)
06-13-03 14:54
No 439768
User Picture 
      P2Ps from benzenes by Friedel-Crafts reaction
(Rated as: excellent)
    

P2Ps and phenylacetonitriles from (un)substituted benzenes by Friedel-Crafts reaction



Chem. Pharm. Bull, 1982, 30(10), 3574-3579 (http://www.angelfire.com/scifi2/lego/journals/2.pdf)


Syntheses of arylacetone and arylacetonitrile by Friedel-Crafts Reaction with alpha-chloro-alpha-(methylthio)-substituted acetone and acetonitrile




[...]



Experimental


alpha-chloro-alpha-(methylthio)acetone (4)
Molecule: A ("C(C(SC)Cl)(C)=O")

This compound was prepared according to the procedure described by Böhme9. N-Chlorosuccinimide (13.9 g, 0.104 mol) was added to a stirred solution of alpha-(methylthio)-acetone10 (10.4 g, 0.1 mol) in carbon tetrachloride (150 ml) in small portions at 0°C and stirring was continued at room temperature for 3 h. The precipitated succinimide was filtered off and and the solvent was removed in vacuo. The residual oil was distilled to give 4 (8.82 g, 64%).

alpha-(methylthio)-phenylacetone (5a)
Molecule: B ("C(C(SC)c1ccccc1)(C)=O")

SnCl4 (1.03 g, 3.96 mmol) was added to a stirred solution of 4 (549 mg, 3.96 mmol) in benzene (9 ml) at 0°C, and stirring was continued at room temperature for 40 min.
The reaction was quenced by the addition of water, and the mixture was extracted with benzene and dried (MgSO4). The solvent was removed in vacuo and the residue was chromatographed on silica gel using benzene as an eluent to give 5a (630 mg, 88%).

alpha-(methylthio)-arylacetones (5b, c, d, e, f, g, h and i): General procedure
SnCl4 or ZnCl2 (for furan) (1.78-1.89 mmol) was added to a stirred solution of 4 (1.78-1.89 mmol) and an aromatic compount [1.78-1.89 mmol except for the case of the reaction of thiophene or furan (2 eq)] in CH2Cl2 (10-15 ml) at 0°C, and stirring was continued under the conditions described in Table 1.
The reaction was quenched by addition of water, then the mixture was extraced with CH2Cl2, and the extract was dried (MgSO4). The solvent was removed in vacuo and the residue was chromatographed using on silica gel benzene as an eluent to give 5b, c, d, e, f, g, h, or i as an oil.

Arylacetones (6a, c, e, f, g, h, and i): General procedure
Zinc dust (1 g) was added to a solution of 5a, c, e, f, g, h, or i (200-300 mg) in acetic acid (2-3 ml), and the resultant mixture was heated with vigorous stirring at 100°C for 1 h, then cooled. Water (20 ml) and CH2Cl2 (30 ml) were added, and the inorganic materials were filtered off. The organic layer was seperated and the aqueous layer was further extracted with CH2Cl2. The combined organic layer was dried (MgSO4) and the solvent was evaporated off. The residue was chromatographed on silica gel using benzene as an eluent to give 6a, c, e, f, g, h, or i as an oil.

alpha-chloro-alpha-(methylthio)-acetonitrile (7)
This compound was prepared according to the procedure described by Böhme9. N-Chlorosuccinimide (11.07 g, 0.083 mol) was added to a stirred solution of alpha-(methylthio)-acetonitrile15 (7.2 g, 0.083 mol) in carbon tetrachloride (45 ml) in small portions at 0°C and stirring was continued at room temperature for 4.5 h. The precipitated succinimide was filtered off and the solvent removed in vacuo. The residual oil was distilled to give 7 (5.52 g, 55%).

alpha-(methylthio)-phenylacetonitrile (8a)
SnCl4 (1.09 g, 4.18 mmol) was added to a stirred solution of 7 (508 mg, 4.18 mmol) in benzene (2 ml) at 0°C, and stirring was continued at room temperature for 40 min. Work-up as described for the preparation of 5a gave 8a (560 mg, 82%).

[...]

alpha-(methylthio)-arylacetonitriles (8c, d, and e): General procedure
SnCl4 or TiCl4 (for 1,2-methylendioxybenzene) (3 mmol) was added to a stirred solution of 7 (3 mmol) and an aromatic compound (3 mmol) in CH2Cl2 (15 ml) at 0°C, and stirring was continued under the conditions described in Table I. The reaction was quenced by the addition of water, then the mixture was extracted CH2Cl2, and dried (MgSO4). The solvent was removed in vacuo and hte residue was chromatographed on silica gel using benzene as an eluent to give 8c, d, or e as an oil.

Arylacetonitriles (9a, c, d, and e): General procedure
Zinc dust (1 g) was added to a solution of 8a, c, d, or e (100-200 mg) in acetic acid (2-3 ml), and the mixture was heated with vigorous stirring at 100°C for 1 h, then cooled. Work-up as described for the preparation of 6 gave 9a, c, d, or e.


Table I. Friedel-Crafts Reactions of aromatic compouds with alpha-chloro-alpha-(methylthio)-acetone (4) and alpha-chloro-alpha-(methylthio)-acetonitrile (7)
4 or 7 ArH ArH/4 or 7a Cat. Temp. Time (min) Product No. Ar in 5 or 8 Yield (%)
4 Benzene b) SnCl4 rt 40 5a   88
4 Toluene 1:1 SnCl4 rt 60 5b c) 89
4 p-Xylene 1:1 SnCl4 rt 60 5c   78
4 Anisole (p-methoxy benzene) 1:1 SnCl4 0°C 45 5d c) 79
4 Anisole (p-methoxy benzene) 1:1 TiCl4 0°C 20 5d c) 47
4 1,2-Dimethoxy benzene 1:1 SnCl4 rt 60 5e   71
4 1,2-Dimethoxy benzene 1:1 TiCl4 rt 60 5e   Trace
4 1,3-Benzodioxole (1,2-methylendioxybenzene) 1:1 SnCl4 0°C 30 5f   70
4 1,3-Benzodioxole (1,2-methylendioxybenzene) 1:1 TiCl4 0°C 60 5f   0
4 Naphthalene 1:1 SnCl4 rt 60 5g   87
4 Thiophene 2:1 SnCl4  0°C 30 5h   45d
4 Furan 2:1 ZnCl2  rt 20 5i   45d
7 Benzene b) SnCl4 rt 40 8a   82
7 Anisole (p-methoxy benzene) 1:1 TiCl4 0°C 60 8b c) 56
7 1,2-Dimethoxybenzene 1:1 SnCl4  0°C 35 8c   60
7 1,3-Benzodioxole (1,2-Methylendioxybenzene) 1:1 TiCl4 rt 75 8d   62
7 Naphtalene 1:1 SnCl4 0°C 60 8e   89

a) Reaction were carried out in methylene chloride unless otherwise indicated
b) Benzene was used as solvent
c) A mixture of o- and p-isomers [...]
d) Yield is based on 4
rt: room temperature

Table II. Yields and spectral data for arylacetones (6) and arylacetonitriles (9)
compd. No. Ar in 6 and 9 Yield (%)
6a Phenylacetone 94
6c 2,5-Dimethylphenylacetone 85
6e 3,4-Dimethoxyphenylacetone 83
6f 3,4-Methylendioxyphenylacetone 82
6g 1-(1-naphthyl)acetone 90
6h 1-thien-2-ylacetone 75
6i 1-(2-furyl)acetone 74
9a Phenylacetonitrile 98
9c 3,4-Dimethoxyphenylacetonitrile 90
9d 3,4-Methylendioxyphenylacetontrile 90
9e 1-naphthylacetonitrile 89


References:
[...]
9) Justus Liebig Ann. Chem., 1977, 51
10) Arg. Biol. Chem., 40, 1031, (1976) and see also references cited herein
[...]
15) Justus Liebig Ann. Chem., 579, 23-27 (1953) (https://www.rhodium.ws/pdf/ch3sch2cn.pdf)
alpha-methylthio-alpha-acetonitrile is prepared from chloroacetonitrile and the sodium salt of methanthiole.





Lego's voice: Workup should bee done by destillation of the crude reaction mixture. The alpha-methylthio-compound is unfortunately not commercially available.

The candle that burns twice as bright burns half as long
 
 
 
 
    Rhodium
(Chief Bee)
06-14-03 12:46
No 439970
User Picture 
      Reference     

Great, here is Ann. Chem., 579, 23-27 (1953) (https://www.rhodium.ws/pdf/ch3sch2cn.pdf) - the other Ann. article will follow after the weekend.
 
 
 
 
    Aurelius
(Hive Addict)
06-15-03 09:03
No 440113
      German     

Rhodium, if you're trying to get me to learn german, right now isn't the time.   It's going to be a bitch to transcribe/translate thatwink  Is it just the same procedure as above?

Act quickly or not at all.
 
 
 
 
    Rhodium
(Chief Bee)
06-16-03 11:04
No 440277
User Picture 
      Don't you worry     

It's all right Aurelius, the german articles are outside of your job description wink Also, they are only considered "further reading" about the pre-precursors and not "required information".
 
 
 
 
    lugh
(Moderator)
07-04-03 23:05
No 444567
      Obscure P2P synthesis
(Rated as: good read)
    

From JCS 127 2349 (1925):

This course of hydrolysis of a coumarin appeared so novel that it was considered desirable to establish it beyond doubt. For this purpose the preparation and hydrolysis of Ghosh's so-called 7-hydroxy-3-phenyl-2-methylbenzo-y-pyrone (which has since been proved to be 7-hydroxy-3-phenyl-4-methylbenzo-a-pyrone, Baker and Robinson, loc. cit.) were repeated, and the formation of benzyl methyl ketone (characterised by means of its semicarbazone and phenylhydrazone), resorcinol, and p-resorcylic acid was confirmed. From 3 g. of this coumarin were obtained about 0.5 g. of benzyl methyl ketone, a slightly smaller quantity of resorcinol, and only a very small amount of p-resorcylic acid, whilst some of the material was unhydrolysed. This confirms the view given above that the principal reaction is the production of benzyl methyl ketone and resorcinol, the p-resorcylic acid being only a by-product, and differs from Ghosh's view, that the chief reaction is the production of benzyl methyl ketone and p-resorcylic acid, the resorcinol being the by-product.
The important conclusion follows from these observations that the only safe criterion of the production of a chromone (I) and not the coumarin (II), is the formation on hydrolysis of a hydroxy­ketone (V), and shows the necessity for extreme caution in deciding the constitution of such substances by hydrolysis. The matter, however, is further complicated in cases where R = methyl (thus including the present), since not only can (I) and (II) give the ketone (III), but also the isomeric coumarin (VI) and the chromone
(VII). As proof of this, the compounds (I) and (VI), having R' _phenyl (Baker and Robinson, loc. cit.), were submitted to hydrolysis under the conditions employed by Ghosh in the case of (II), the main results of which were verified as described above. Compound I underwent complete hydrolysis and gave a small amount of benzyl methyl ketone and about a 70% yield of 2:4-dihydroxyphenyl benzyl ketone ; whilst compound VI, though incompletely hydro­lysed, gave a relatively large yield of benzyl methyl ketone and resorcinol. The benzyl methyl ketone was in all cases characterised by means of its semicarbazone and phenylhydrazone, and the pelting points of mixtures of any of the specimens showed no depression. Compound VII (Hannach and Kostanecki, Ber., 1902, 35, 866) has not actually been hydrolysed under these conditions, but there is no doubt that it also would give benzyl methyl ketone as one of its fission products.
smile

Chemistry is our Covalent Bond
 
 
 
 
    Lego
(Hive Bee)
07-21-03 15:54
No 448826
User Picture 
      P2Ps via Meerwein arylation Actually Lego was...
(Rated as: excellent)
    

P2Ps via Meerwein arylation



Actually Lego was searching for new methods to synthesize indoles but in Journal of Organic Chemistry (1983), 48(12),  2066-9 2-nitroanilines are diazotized, then treated with isopropenylaceate and the reduced/cyclized to indole. The authors mentioned that the intermediate is 2-nitro-P2P!

So some more literature references were searched to make sure that aniline can bee used as a precursor for P2P (btw: substituted anilines work, too).

Unfortunately none of the found methods was used with plain aniline but with substituted anilines. But exchanging vinylacetate for isopropenylacetate yields phenylacetaldehydes with the same mechanism and in Journal of General Chemistry (Translation of Zhurnal Obshchei Khimii), (2000), 70(10), 1600-1602 (http://angelfire.lycos.com/scifi2/lego/journals/24.djvu) the authors use plain aniline to synthesize phenylacetaldehyde, therefore Lego sees no reason why this reaction should not work with unsubstituted aniline.

This methods seems to bee useful for converting any (un)substituted aniline via diazotiation into phenylacetaldehyde, phenylaceton or higher 2-phenylketones.


See Patent US5811586 for the synthesis of 3-(trifluormethyl)-P2P.

In Patent EP1137627 2,4-difluoro-P2P is prepared in 82.9% yield.


In Bull.Soc.Chim.Fr.; 1972; 1926-1931 (http://angelfire.lycos.com/scifi2/lego/journals/1.pdf)4-chloro-P2P is prepared according to the following procedure (translated by Chimimanie, thanks!).


4-Chloro-phenylacetone 20

Excess of enol acetate
A solution of the isopropenyl acetate 2 (5 mL) in acetone (60 mL) is treated by the diazonium salt (1g p-chloroaniline in 20 mL water) in presence of the standard catalyst (25 mL).
The p-Chlorophenylacetone 20 (580 mg) is elued with a 80-20 petroleum ether-et2o mixture. Yield: 580 mg

Excess of diazonium salt
A solution of isopropenyl acetate 2 (390 mg) in acetone (50 mL) is treated by the diazonium salt (1g p-chloroaniline, 20 mL H2O) in the presence of the standard catalyst (25 mL).
The mixture 80-20 petroleum ether-et2o elute the p-chlorophenylacetone 20 (65mg) identified with an authentical sample.

Preparation of the aqueous solution of the p-chlorobenzenediazonium chloride
To a solution of p-chloroaniline (1 equivalent) in a water-HCl 12N (3 equivalents) mixture (the quantity of water is indiqued in each case), is added a concentrated aqueous solution of sodium nitrite, the temp being inferior to 5°C. The addition is stopped when a positive reaction is displayed by a iodo-amidonned paper, then a saturated aqueous solution of sodium acetate is added to bring pH to 3.


Catalyst:

Preparation of the standard catalyst.

This is an aqueous solution of cuprous chloride Prolabo (5g) in the melange acetone (160 mL)-water(80 mL), additioned of chlorhydric acid 12N till disoolution of the cupprous chloride (~4mL), the melange is then degassed with azote.


General modus operandi.

To a solution of the organic substrate in acetone, degassed with azote, is added the aqueous solution of the diazonium salt, then the standard catalyst.

Stirring is applied till end of the gaseous evolution, then 1h more, then the acetone is evaporated and, after the habitual treatment, the crude product is chromatographied on a silica column (100 times the weight of the crude product).

Petroleum ether elute the p-dichlorobenzene, then a mixture of petroleum ether-et2o elute the products of the reaction (cfr each case).





All quoted methods vary slightly but generally the aniline is diazotized, the acidic solution is buffered and isopropenylacetate/vinylacetate is added or the the diazotized aniline is added to the enolacetate and the reaction is extracted with NP, evaporated and purified either by distillation or chromatography.

Vinylacetate and isopropenylacetate are commercially available and cheap. Sodium nitrite can bee prepared OTC, see http://www.geocities.com/dritte123/NaNO2.html) and anilines can either bee purchased (Chimimanie just mentioned that aniline is watched as a precursor for fentanyl synthesis) or prepared via nitration and reduction.


The candle that burns twice as bright burns half as long
 
 
 
 
    Rhodium
(Chief Bee)
07-22-03 00:08
No 448923
User Picture 
      Nucleophilic radical alkylation of diazonium salts     

Great post Lego! I have seen that reaction before, but those papers are still in my rather large pile of "interesting articles I haven't had time to post yet", but I can say right away that you have found a few of the best ones already.

A very similar reaction is Post 392827 (Rhodium: "Aniline to Allylbenzene by Diazotization (Diazall)", Methods Discourse) where they diazotize anilines and react the diazonium salts with allyl bromide to form the corresponding allylbenzenes in up to 85% yield.



Unfortunately, both in this allylation and in the Meerwein Arylation, the yields drop if the reaction is performed on an electron-rich substrate (in the Diazo-Allylation linked above, they get only ~20% yield of 4-MeO-allylbenzene). I guess this means that we can probably abandon the idea of a high-yielding synthesis of safrole/MDP2P from 3,4-MD-aniline right away... frown
 
 
 
 
    Aurelius
(Active Asperger Archivist)
07-22-03 14:33
No 449058
      Preparation of Sodium Nitrite inorg. chem. prep.
(Rated as: excellent)
    

Preparation of Sodium Nitrite

Inorganic Chemical Preparations, Erdmann & Dunlap© 1900

From PolytheneSam’s web site

Abstract:

The reduction of sodium nitrate with lead to produce lead oxide and sodium nitrite.


General Procedure:

After removing the small dome from the inner fireclay mantle of a Rössler gas-furnace, place upon the mantle a strong iron dish (2.5cm high, 12 cm upper diameter) having a smooth bottom.  Place 85g of chili saltpeter (sodium nitrate) in the dish and close the furnace.  As soon as the dish has become faintly incandescent and the molten nitrate just begins to give off bubbles of oxygen, gradually add 206g of lead in the form of old pieces of sheet lead or lead tubing.  The lead is at once vigorously oxidized, and, if stirred continually with an iron spatula, becomes almost completely into its oxide in half an hour.  Empty the contents of the small iron dish into a large deep iron one, and repeat the operation several times, using the same amounts of sodium nitrate and lead.  Place the various products in the large iron dish, extract once with boiling water, and decant upon a creased filter.  Dry the residue of lead oxide and set it aside for other experiments (exp on page 52).  Pass a strong current of carbon dioxide into the still boiling-hot filtrate, for a few minutes only; filter off the lead carbonate which separates, and neutralize the solution while stirring, by carefully adding nitric acid by pipette or burette.  Evaporate the solution to crystallization.  The crystals which separate first, consist partly of nitrate and may be used again for remelting with lead; the mother liquor gives pure nitrite.  A normal solution of the nitrite is prepared by dissolving 69g of it into water and diluting to one liter.

Reaction:

NaNO3 + Pb = NaNO2 + PbO

A small part of the nitrite is converted into sodium plumbate, which must be decomposed by carbon dioxide.—Nitrous acid reacts with peroxides and with permanganate according to the following equations:

MnO2 + HNO2 + HNO3 = Mn(NO3)2 + H2O
2HmnO4 + 5HNO2 = 2Mn(NO3)2 + HNO3 +3H2O


with sulphanilic acid it gives diazobenzenesulphonic acid.

Testing:

Sodium nitrite crystals, upon treatment with dilute mineral acids, go very easily into solution, with considerable effervescence and evolution of nitrous anhydride.  The acid solution turns potassium iodide and starch paper blue and dissolves the peroxides of manganese and lead with great rapidity.  To test the strength of the sodium nitrite, fill a burette with the normal solution and run it into a solution of 11.55g of the sodium salt of sulphanilic acid, which is cooled by ice and made strongly acid with hydrochloric acid; stir constantly, and continue the addition of the nitrite solution until a drop of the liquid gives a strong blue color with potassium iodide and starch paper.  If the nitrite is pure, 50cc are necessary.  Or 50cc of the nitrite solution may be measured into a graduated flask by means of a pipette, diluted to one liter, and this dilute solution run from a burette into tenth normal potassium permanganate which is made with sulfuric acid and warmed to 40-50*C, stirring during the addition.  Just 50cc of the twentieth normal nitrite solution should be used for 50cc of the permanganate.

Detection of Nitrous Acid:

(i) on adding dilute sulphuric acid, potassium iodide, and a solution of starch to a nitrate, a blue color develops, by liberation of free iodine, which combines with the starch.  Nitrates produce a similar color only after standing for some time.
(ii) Ferrous sulphate and acetic acid produce a brown color which is discharged on warming, brown fumes being evolved (compare detection of nitric acid, p 453)
(iii) Meta-phenylenediamine in acid solutions gives a brown color with nitrous acid

Estimation of Nitrous Acid and Nitrites:

(i) Nitrites can be estimated by reduction to nitric oxide as in the case of nitric acid.
(ii) When present in smaller quantities, the nitrite may be reduced to ammonia by means of a zinc-copper couple and the ammonia estimated by Nessler’s solution or by titration as in the case of nitric acid
(iii) In the absence of other oxidizable compounds, nitrous acid (e.g., in “waste acid” from nitration) can be estimated by oxidation with potassium permanganate,
(iv) Traces of nitrites may be estimated by means of the color produced with meta-phenyenediamine.


2KMnO4 + 3H2SO4 + 5HNO2 = K2SO4 + 2MnSO4 + 5HNO3 + 3H2O

Act quickly or not at all.
 
 
 
 
    viki
(Stranger)
07-23-03 03:28
No 449141
      aldol condensation-Baeyer Villiger oxidation     

Has anyone performed the two dogs proposal set out in post Post 392078 (Aurelius: "P2P synth", Methods Discourse) ?
Feedback would be deeply appreciated.Cheers Viki

viki
 
 
 
 
    Lego
(Hive Bee)
10-14-03 22:49
No 464635
User Picture 
      P2P from bromobenzene/CuI/Acetylacetone
(Rated as: excellent)
    

Chemistry Letters, 1982, 597-600

A versatile synthesis of arylacetones from aryl halides and acetylacetonate


Saburo Sugai*, Hiroshi Ikawa, Tokuji Okazaki, Sanya Akaboshi, and Shiro Ikegami

Research Laboratories, Ohta Pharmaceutical Co. Ltd., Namiki, Kawaguchi 332, and
Faculty of Pharmaceutical Sciences, Teikyo University, Sagamiko, Kanagawa 199-01



Reaction of aryl halides with sodium or potassium acetylacetonate in the presence of a copper catalyst affords directly arylacetones resulted from deacetylation of initially formed 3-arylacetylacetones in good yields.


An investigation of a new efficient synthetic method of arylacetones gives us interesting useful intermediates for drug synthesis of central nervous agents. In this paper, we wish to report an efficient, direct synthesis of arylacetones from the reaction of aryl halides with acetylacetone.

Some preparations of arylacetones have been reported1-4. However, these reactions mostly need drastic conditions and a few or more steps, and generally afford the corresponding products only in moderate or poor yields. Hence, particularly, in manufacturing scale, they would be less useful. Thus, we were interested in the utility of acetylacetone as a reagent convertible easily to an acetonyl group and studied reactions with aryl halides.

The reaction of acetylacetone with aryl halides has been hitherto known only in cases of aryl halides substituted with a carboxy group in the ortho-position in the presence of Cu(II) derived from disproportionation of Cu(I)5-6. In such a case, it was presumed that a carboxylate anion would be concerned in reaction5.

We investigated the direct arylation of acetylacetone with aryl halides without the aid of such a carboxyl group. Consequently, we could obtain 3-arylaetones from the reaction in the presence of Cu(I) halide. Since these 3-arylacetylacetones trend to deacetylate under alkaline condition, subsequent deacetylation in diluted alkaline solution in the same pot afforded the corresponding arylacetones in good yields. This procedure must be simple because two steps reaction can be performed in a one-pot.

ArX (1) 1 equiv. + M+CH(COCH3)2 (2) 5-10 equiv. M = Na, K --CuY (3) 1-2 eq.mol/DMF, 100-120°, N2--> ArCH(COCH3)2 (4) --OH--(-COCH3)--> ArCH2COCH3 (5)

A typical procedure is as follows: 3-bromobenzophenone (5 mmol) was dissolved in 15 ml of dry dimethylformamide, then CuI (5 mmol) and potassium acetylacetonate (25 mmol)7 were added successively under a nitrogen current. The reaction mixture was heated for 6 hr at 100°C to afford a mixture of acetylbenzophenone, bp1.0 177-181 °C; vC = 0 1715, 1650; d in CDCl3 2.16 (3H, s, CH3), 3.75 (2H, s, CH2), 7.30-7.85 (9H, m, aromatic); M+ 238, and 3-(3-benzoylphenyl)acetylacetone, vC = 0 1650, 1600; d in CDCl3 1.86 (6H, s, CH3x2), 7.30-7.80 (10 H, m, aromatic and CH); M+280, in 60 and 14% yields, respectively. The latter acetylacetone compound was quantitatively converted to 3-acetonylbenzophenone by treatment with diluted NaOH aq. solution at room temperature. After usual work-up, the products were purified by silica-gel column chromatographic technique.

Usually this procedure could be successively performed in the same reaction vessel without isolation of both compounds. The results obtained using various aryl halides under several different conditions are summarized in Table I.

Table 1
Entry ArX (1) M (2) Y (3) Time (hr) 4a
Yield (%)b
5a
Yield (%)b
1 3-PhCOC6H4Br K I 6.0 14c 60c
2 3-PhCOC6H4Br K Br 7.5   80
3 3-PhCOC6H4Br K Cl 11.0   76
4 3-AcC6H4Br Na I 15.0   81
5 4-AcC6H4Br Na I 15.0   66
6 4-NO2C6H4Br Na I 1.5   43
7 3-F-4-PhC6H44Br Na I 14.0   67
8 PhBr Na I 30.0   72
9 2-CH3C6H4Br Na I 48.0   32
10 4-CH3C6H4Br Na I 23.0 10c 65c
11 4-CH3C6H4Br K I 9.0   85
12 4-CH3C6H4I K I 2.0   82
13 4-CH3OC6H4Br Na I 23.0 26c 67c
14 4-AcNHC6H4Br Na I 27.0   82d

a) Mp and bp of new product; Entry 7 (5); mp 74.5-75.5°C, Entry 13 (4); bp7.0130-134°C
b) Isolated yield
c) Yield resulted from the initial reaction is shown
d) Used 2 equiv. of 3

As shown in Table I, it is indicated that the combination of cuprous iodide and potassium acetylacetonate gives the best result.

In this reaction, the differences of reaction conditions in changes of solvent, temperature, and catalyst would seriously affect the formation of product. The use of a protic solvent did not result in the formation of the desired product, but afforded the corresponding reduced product as a major product. Therefore, aprotic solvents having high solubility such as dimethylformamide should be desired in this reaction.

The reaction temperature over 120°C should be avoided because the decomposition of acetylacetonate might be caused.

Next, we found that the reaction proceeded not only in the presence of Cu(I) halides as catalyst, but also in the reaction with Cu(II) acetylacetonate resulted from disproportionation. The catalytic abilities of Cu(II) and Cu(0) were also investigated for exploiting the reaction process.

Based on the information that the formation of Cu(0) and Cu(II) acetylacetonate have been anticipated in our system, we investigated these catalytic abilities for the substitution reaction of acetylacetone with 3-bromobenzophenone. The reaction of 3-bromobenzophenone (1 eq. mol) with or without potassium acetylacetonate (4 eq. mol) was carried out in dimethylformamide at 100°C in the presence or absence of Cu(0), Cu(I) or Cu(II) as a catalyst. After the reaction, 3-(3-benzoylphenyl)acetylacetone formed was successively deacetylated by treatment with diluted NaOH aq. solution in the same vessel. These results are summarized in Table II.


Reaction scheme



Table II
Run Catalyst (Molar Ratio) Time (hr) Yield (%)a of 5a
1 Cu2+(CHAc2)2 (1) 24 no reactionb
2 --- 24 tracec,d
3 Cu2+(CHAc2)2 (0.5) 11 61
4 Cu2+(CHAc2)2 (0.5) + Cu(0) (0.5) 11 61
5 Cu2+(CHAc2)2 (0.5) + KI (1) 7 77
6 CuI (1) 20 74e
7 Cu(0) (0.1) 20 traced

a) Isolated yield
b) In the absence of 2
c) In the absence of a catalyst
d) Detected by TLC and GLC
e) Used 5 equiv. of 2

The substitution reaction could not completely take place with only Cu(II) acetylacetonate formed by disproportionation (Run 1). Furthermore, 3-bromobenzophenone was hardly substituted with only use of potassium acetylacetonate (Run 2). In addition, Cu(0) exhibited littly catalytic activity (Run 7). Apparently, in the promotion of the present reaction, the presence of Cu(I) and/or Cu(II) would be essential. The catalytic activity of Cu(I) appears to be equal or greater than that of Cu(II), and the addition of potassium iodide may accelerate the rate due to the activation of Cu(II).

An application to the synthesis of drugs was attempted as follows. For example, as shown in the following sheme, 2-(3-benzoylphenyl)propionic acid (7), a potent anti-inflammatory agent, could be obtained in 88% yield from 5a in only two steps8.

Reaction scheme

The present, new efficient synthetic method of arylacetones would provide a some class of synthetic intermediate useful for drug syntheses.



References
1) a) R.M. Hebst and R.H. Manske, Org. Synthesis, 16, 47(1936).
b) J.M.Potapov and A.T.Terenteo, Zh. Obsch. Khim., 28, 3323(1958).
2) C. Tegner, Acta Chem. Scand., 6, 782(1952)
3) M.E. Dullaghan and F.F. Nord, J. Org. Chem., 18, 878(1953)
4) J. Smidt, W. Hafner, and R. Jiva, Patent US3080425, 1963
5) R.G.R. Bacon and J.C.F. Murray, J. Chem. Soc., Perkin Trans. 1, 1975, 1267.
6) R. Nast, R. Mohr, and C. Schultze, Chem. Ber., 96, 2127(1963).
7) Sodium and potassium acetylacetonate prepared from the reaction of acetylacetone with an equimolar amout of NaOH or KOH dissolved in ethanol, followed by drying the isolated salts under reduced pressure, were used. Commercially available cuprous halides were directly used without further treatment.
8) A part of this work was presented at at the 101st Annual Meeting of Pharmaceutical Society of Japan, Kumamoto, April 1981.


https://www.rhodium.ws/djvu/p2p-acetylacetonate.djvu
https://www.rhodium.ws/chemistry/p2p.acetylacetonate.html

The candle that burns twice as bright burns half as long
 
 
 
 
    Rhodium
(Chief Bee)
11-14-03 02:05
No 470750
User Picture 
      Cognate Preparation
(Rated as: excellent)
    

Copper-catalyzed reaction of aryl iodides with active methylene compounds
Kazumi Okuro, Makoto Furuune, Masahiro Miura, Masakatsu Nomura
J. Org. Chem. 58, 7606-7607 (1993) (https://www.rhodium.ws/pdf/cu-cat.arylation.acac.pdf)

Synthesis of Phenyl-2-Propanone:



Iodobenzene (4g, 20 mmol) was reacted with acetylacetone (4g, 40 mmol), CuI (380 mg, 2 mmol) and K2CO3 (11g, 80 mmol) in DMSO (50 mL) at 120°C under N2 for 4h. The resulting mixture was poured into dilute hydrochloride acid, extracted with ether, and dried over sodium sulfate, yielding 3-phenyl-2,5-pentanedione (65%) and phenyl-2-propanone (11%), which could be purified by flash chromatography. Stirring the crude reaction mixture with dilute aqueous sodium hydroxide at room temperature effected deacetylation, giving phenyl-2-propanone in 70-75% yield.
 
 
 
 
    pHarmacist
(pHantasticant)
11-14-03 02:38
No 470759
User Picture 
      Aaah, a nice one!     

Great find there, Rhodium! Could this become the prefered route to P-2-P's? It sure looks like that! It can't get much more simple that this, it will bee impossible to controll this compound...

http://www.bassdrive.com/BassDrive.m3u
 
 
 
 
    Rhodium
(Chief Bee)
11-14-03 02:47
No 470762
User Picture 
      Various halobenzenes     

Note that the procedure in Post 464635 (Lego: "P2P from bromobenzene/CuI/Acetylacetone", Methods Discourse) uses bromobenzenes, which are considerably cheaper than iodobenzenes. Didn't you realize that the previous procedure was better than the one I posted? Imagine what a difference a little reaction diagram makes in education...
 
 
 
 
    pHarmacist
(pHantasticant)
11-14-03 02:51
No 470763
User Picture 
      Actually I didn't really look at the one Lego...     

Actually I didn't really look at the one Lego posted at all, as normally, P-2-P don't interest me, but your little reaction diagram did couch my eye. Hmm, I'll look at Lego's post now, wink

http://www.bassdrive.com/BassDrive.m3u
 
 
 
 
    Aurelius
(Active Asperger Archivist)
11-14-03 04:33
No 470781
      PDF     

The PDF isn't pulling up properly in my browser.  Is the link broken?  something else? 

In any case, What's the rxn times for each step?

Act quickly or not at all.
 
 
 
 
    Rhodium
(Chief Bee)
11-14-03 14:44
No 470855
User Picture 
      rxn times     

The latest PDF shows for me, do you have the latest (6.0) Acrobat Reader installed?

The reaction times for the formation of the 3-phenyl-acetylacetone varies with the substrate, but is 4h with PhI and 30h with PhBr. The deacetylation is described as follows: The [3-phenylacetylacetone] was quantitatively converted to [the corresponding phenylacetone] by treatment with diluted NaOH aq. solution at room temperature.
 
 
 
 
    bio
(Newbee)
11-14-03 19:44
No 470893
      "P2P compilation"     

Hey Aurelius; Why don't you put up the twodogs procedure for PhCHO&MEK to P2P? It really does work. My last yield on the methyl phenyl butenone was 19% better than published on half kilo scale. Somebee told me this may be the only OTC phenylacetone.
 
 
 
 
    Rhodium
(Chief Bee)
11-17-03 17:30
No 471395
User Picture 
      Basic Deacetylation of Acetylacetones
(Rated as: good read)
    

Mechanism of the Hydrolytic Cleavage of Carbon-Carbon Bonds. I. Alkaline Hydrolysis of beta-Diketones
By Ralph G. Pearson & Evan A. Mayerle
J. Am. Chem. Soc. 73, 926-930 (1951) (https://www.rhodium.ws/pdf/beta-diketone.cleavage.mechanism.pdf)

Abstract
The rates of alkaline hydrolysis of acetylacetone, methylacetylacetone and dimethylacetylacetone have been studied over wide range of concentrations, and the “true” rate constants for the reactions have been calculated. The mechanism appears to involve reactions of the neftral diketone molecule with either one or two hydroxide ions for the “acid” and acetylacetone and methylacetylacetone, but with only one hydroxide ion for dimethylacetylacetone. “Ketone” cleavages of acetoacetic ester are commented upon.
 
 
 
 
    lugh
(Moderator)
12-07-03 00:43
No 475103
      Another Obscure P2P Synthesis
(Rated as: good read)
    

Another obscure p2p synthesis, using cyclohepta­triene; from C R Acad Sc Paris 272 1664-6 (1971):



cool

Chemistry is our Covalent Bond
 
 
 
 
    Lego
(Hive Bee)
12-13-03 17:52
No 476621
User Picture 
      Phenylacetic acid to P2P w/o acetic anhydride
(Rated as: excellent)
    

A New Method for Expeditious Ketone Synthesis from Acids via Acyl Hemiacetals

Matthew N. Mattson and Henry Rapoport
J. Org. Chem., 1996, 61(17), 6071-6074 (http://lego.chemistry.tripod.com/p2p.enolether.html)
DOI:10.1021/jo9607471




...

2-Tetrahydrofuranyl 3-Phenylpropionate (8). To a stirred solution32 of 3-phenylpropionic acid (200 mmol) and 2,3-dihydrofuran (210 mmol) in dichloromethane (200 mL) at -40 °C was added a solution of methanesulfonic acid (195 µL, 10% v/v in CH2Cl2, 0.15 mol %, 0.30 mmol). The stirred reaction mixture was warmed over 30 min to 0 °C, stirred for 4 h, and then recooled to -40 °C. A solution of dry 1,2-ethylenediamine33 (800 µL, 10% v/v in CH2Cl2, 0.60 mol %, 1.2 mmol) was added, and the reaction mixture was warmed to 10 °C and placed in a separatory funnel containing dichloromethane (450 mL). The dichloromethane solution was washed with chilled bicarbonate solution (2 Ч 200 mL, 1/1/1 H2O/brine/saturated NaHCO3, 5 °C) and brine (2 Ч 100 mL), dried (Na2SO4), evaporated in vacuo (40 °C bath), and chased by dilution and evaporation with hexanes (2 Ч 60 mL) to yield 8 as a clear oil (43.54 g, 99% yield)

Reaction of THF Esters with Grignard Reagents. General Procedure. To a stirred solution of the THF ester (10.0 mmol) in dichloromethane (30 mL) at -20 °C under nitrogen was added dropwise over 5 min the organomagnesium reagent (in Et2O or THF, 1-3 M, 10.0 mmol) to yield a clear lightly colored solution. After 30 min at -20 °C, the reaction mixture was slowly warmed to 18 °C over 18 h and then recooled to 0 °C during dropwise addition with stirring into 1.0 M H3PO4 (14 mL) at 0 °C.34 The mixture was warmed to 20 °C, diluted with water (10 mL), and added to a separatory funnel using dichloromethane (50 mL). The organic layer was separated, and the aqueous phase was extracted (CH2Cl2, 3 Ч 10 mL). The combined organic phases were shaken with 0.25 M H3PO4 (12 mL) for 5 min, separated,35 washed with aqueous NaHCO3 (3 Ч 20 mL, 1/1, H2O/saturated NaHCO3) and brine (25 mL), and then dried (Na2SO4) and evaporated (up to 40 °C). Dilution and evaporation with hexanes (2 Ч 60 mL) yielded the crude ketone as an oil. Chromatography on silica gel using pentane/Et2O, when necessary, provided the pure ketone.

1-Phenyl-3-pentanone (9a).36 The above procedure was employed using ethylmagnesium bromide to yield 289 mg (88%) of a clear oil.

...

32. When the carboxylic acid was not completely soluble at -40 or -20 °C, addition of the methanesulfonic acid to the suspension catalyzed a slow clarification to form a solution of the more soluble THF ester.
33. Dry amine quenches (dilute ammonia, triethylamine, or 1,2-ethylenediamine in CH2Cl2) at low temperature gave better results than an aqueous alkaline quench (NaHCO3).
34. Quenching by bolus addition of the quench into the vigorously stirred reaction mixture gave similar results. The ketone 9g was quenched with pH 7, 0.1 M phosphate buffer.
35. If the THF ester is still present (TLC), the CH2Cl2 solution is stirred with methanesulfonic acid and methanol (e.g., 5 µL and 1 mL, respectively), at 20 °C until hydrolyzed (1 h).
36. Schultz, E. M.; Bicking, J. B. J. Am. Chem. Soc. 1953, 75, 1128.





Although the authors do not synthesize P2P with methylmagnesiumbromide they prove the synthetic abilities of this method with ethylmagnesiumbromide which yields a precursor for aephetamine.

The candle that burns twice as bright burns half as long
 
 
 
 
    ragnaroekk
(Hive Bee)
12-15-03 14:53
No 476998
      cycloheptatrien     

aka tropylium aka  cycloheptatrienylium (IUPAC)

cycloheptatriene is rather uncommon.

Honi soit qui mal y pense
 
 
 
 
    hypo
(Hive Addict)
12-15-03 15:02
No 476999
User Picture 
      ???     

google:

cycloheptatriene: 800 hits
tropylium: 700 hits
cycloheptatrien: 100 hits
cycloheptatrienylium: 20 hits

filter(lambda W : W not in 'ILLITERATE','BULLSHIT')
 
 
 
 
    Rhodium
(Chief Bee)
12-15-03 18:58
No 477026
User Picture 
      cycloheptatriene and its tropylium anion     

aka tropylium aka  cycloheptatrienylium (IUPAC)

That is the aromatic anion of cycloheptatriene. Big difference.

The Hive - Clandestine Chemists Without Borders
 
 
 
 
    Rhodium
(Chief Bee)
01-16-04 08:38
No 482763
User Picture 
      P2P syntheses of yonder days
(Rated as: excellent)
    

Ueber den alpha-Phenylacetessigester
Walter Beckh
Chem. Ber. 31, 3160-3164 (1898) (https://www.rhodium.ws/pdf/p2p.phenylacetoacetonitrile.pdf)

Summary:
Sodium ethoxide catalyzed condensation of ethyl acetate with phenylacetonitrile, forming alpha-Phenylacetoacetonitrile, which upon treatment with sulfuric acid is simultaneously hydrolyzed to the acid and decarboxylated to give phenylacetone.
____ ___ __ _

Ueber Phenacetylmalonsäureester
Hermann Metzner
Ann. 298, 374-378 (1897) (https://www.rhodium.ws/pdf/p2p.phenylacetylmalonate.pdf)



Summary:
Phenylacetic acid is chlorinated with PCl5 in chloroform, yielding phenylacetyl chloride in 88% yield. This is then used to acylate sodium diethylmalonate to give the phenylacetylmalonic ester in 75% yield by leaving the mixture undisturbed for a week. Refluxing the phenylacetylmalonic ester in 20% (1.1 g/mL) aqueous hydrochloric acid for a few hours results in double ester hydrolysis and decarboxylation, leaving phenylacetone as the end product.
____ ___ __ _

Oxydationsprodukte der Benzylketone
A. Popoff
Chem. Ber. 5, 500-502 (1872) (https://www.rhodium.ws/pdf/p2p.me2zn.pdf)

Summary:
Alkylation of phenylacetyl chloride with dimethyl zinc under cooling, followed by hydrolysis of the intermediate with dilute hydrochloric acid gave crude P2P, bp 210-217°C. After purification via the crystalline bisulfite adduct, the ketone had bp 214-216°C.
____ ___ __ _

Zur Geschichte der Phenylessigsäure
B. Radziszewski
Chem. Ber. 3, 198-199 (1870) (https://www.rhodium.ws/pdf/p2p.paa-ba-acetate.pdf)

Summary
Phenylacetone is prepared by pyrolysis of phenylacetic acid and barium acetate (1:1, w/w). The distillate also contains acetone, toluene and diphenylacetone. The phenylacetone is then isolated by fractional distillation, bp 215°C, d. 1.010 g/mL at 3°C.

The Hive - Clandestine Chemists Without Borders
 
 
 
 
    Rhodium
(Chief Bee)
01-20-04 23:32
No 483632
User Picture 
      Enzymatic Oxidation of Propenylbenzene to P2P     

Monooxygenase activity of cytochrome c peroxidase
VP Miller, GD DePillis, JC Ferrer, AG Mauk and PR Ortiz de Montellano
J. Biol. Chem. 267, 8936-8942 (1992) (http://www.jbc.org/cgi/reprint/267/13/8936)

Abstract
Recombinant cytochrome c peroxidase (CcP) and a W51A mutant of CcP, in contrast to other classical peroxidases, react with phenylhydrazine to give sigma-bonded phenyl-iron complexes. The conclusion that the heme iron is accessible to substrates is supported by the observation that CcP and W51A CcP oxidize thioanisole to the racemic sulfoxide with quantitative incorporation of oxygen from H2O2. Definitive evidence for an open active site is provided by stereoselective epoxidation by both enzymes of styrene, cis-beta-methylstyrene, and trans-beta- methylstyrene. trans-beta-methylstyrene yields exclusively the trans- epoxide, but styrene yields the epoxide and phenylacetaldehyde, and cis- beta-methylstyrene yields both the cis- and trans-epoxides and 1-phenyl- 2-propanone. The sulfoxide, stereoretentive epoxides, and 1-phenyl-2- propanone are formed by ferryl oxygen transfer mechanisms because their oxygen atom derives from H2O2. In contrast, the oxygen in the trans- epoxide from the cis-olefin derives primarily from molecular oxygen and is probably introduced by a protein cooxidation mechanism. cis-[1,2-2H]- 1-Phenyl-1-propene is oxidized to [1,1-2H]-1-phenyl-2-propanone without a detectable isotope effect on the epoxide:ketone product ratio. The phenyl-iron complex is not formed and substrate oxidation is not observed when the prosthetic group is replaced by delta-meso-ethylheme. CcP thus has a sufficiently open active site to form a phenyl-iron complex, to oxidize thioanisole to the sulfoxide, and to epoxidize styrene and beta-methylstyrene.

The Hive - Clandestine Chemists Without Borders
 
 
 
 
    Rhodium
(Chief Bee)
01-26-04 05:03
No 484608
User Picture 
      Phenylacetic Acid -> P2P [ ThO2 Tube Furnace ]
(Rated as: excellent)
    

Aerogel Catalysts
Thoria: Preparation of Catalyst and Conversion of Organic Acids to Ketones

S. Swann, E. G. Appel, S. S. Kistler
Ind. Eng. Chem. 26(4), 388-391 (1934) (https://www.rhodium.ws/pdf/p2p.thoria-1.pdf)

Abstract
The conversion of aliphatic Acids to ketones has been studied over thoria aerogel. It has been found that the aerogel catalyst is distinctly superior to thoria hydrogel, thoria prepared from the oxalate, and thoria on pumice for this purpose. The yields of ketones compare favorably with the best reported in the chemical literature.
____ ___ __ _

Thoria Aerogel Catalyst: Aliphatic Esters to Ketones
S. Swann, E. G. Appel, S. S. Kistler
Ind. Eng. Chem. 26(9), 1014 (1934) (https://www.rhodium.ws/pdf/p2p.thoria-1.pdf)

The Hive - Clandestine Chemists Without Borders
 
 
 
 
    Rhodium
(Chief Bee)
04-24-04 23:02
No 502749
User Picture 
      Phenylacetyl chloride + (CH3)2Cd -> P2P
(Rated as: excellent)
    

Alkyl Benzyl Ketones and Hydantoin Derivatives
E.H. Sund and H.R. Henze
Journal of Chemical and Engineering Data 15(1), 200-201 (1970)

Ten alkyl benzyl ketones were synthesized by the interaction of phenylacetyl chloride and the requisite dialkyl cadmium, the synthesis being modeled after a published procedure by Blaise [Compt. Rend. 133, 1218 (1901)].

Experimental
Preparation of Phenyl-2-Propanone

A mixture of 40 ml. of anhydrous ether and 6.1 grams (0.25 mole) of magnesium was stirred under reflux while 35.5 grams (0.25 mole) of methyl iodide in 140 ml. of anhydrous ether was added over a 3-hour period; stirring under reflux was continued for an additional hour. The reaction mixture was cooled with an ice bath and 22.4 grams (0.134 mole) of powdered anhydrous cadmium chloride was added over a 5- to 10-minute period, warmed to room temperature, and refluxed on a steam cone for 1 hour. Ether was removed by distillation on a steam bath. To theresidue was added 100 ml. of anhydrous benzene and the distillation was continued until about 50 ml. more of distillate was collected. Again 100 ml of anhydrous benzene was added, the flask was cooled in an ice bath, and 30.9 grams (0.2 mole) of phenylacetyl chloride in  75 ml of anhydrous benzene was added with stirring over a period of approximately 10 minutes. The reaction mixture was warmed to room temperature and refluxed with stirring on a steam cone for 1 hour. The flask was again cooled in an ice bath and the reaction mixture decomposed by the addition of a solution of 25 grams of ammonium chloride in 200 ml of cold water. The organic phase was separated, washed, and dried over anhydrous sodium sulfate. The benzene was removed by flash distillation and the ketone distilled under reduced pressure. There was thus obtained 15.5 grams (58%) of 1-phenyl-2-propanone, bp 74-76°C/3 mmHg).

The Hive - Clandestine Chemists Without Borders
 
 
 
 
    Rhodium
(Chief Bee)
04-26-04 16:32
No 503140
User Picture 
      P2P by acylation of diethyl malonate
(Rated as: excellent)
    

Synthesis of Methyl Ketones from Diethyl Acylmalonates
Howard G. Walker and Charles R. Hauser
J. Am. Chem. Soc. 68, 1386-1388 (1946) (https://www.rhodium.ws/pdf/p2p.diethylacylmalonate.pdf)

A convenient method for preparing certain methyl ketones consists in the acylation of the sodium or, preferably, the magnesium-ethoxy derivative of diethyl malonate with the appropriate acid chloride, followed by hydrolysis and decarboxylation of the two ester groups of the resulting diethyl acylmalonate in the presence of acid, thus:



In the present investigation, satisfactory yields [was obtained of] phenylacetone1. We have chosen the magnesium ethoxy derivative of diethyl malonate2 rather than the sodium derivative because we believe the former is prepared more conveniently. Although the acid chloride does not appear to react appreciably with the excess alcohol2 used in the preparation of the magnesium-ethoxy derivative, we have employed a 10% excess of the latter in order to minimize this possible side reaction. The crude diethyl acylmalonates were hydrolyzed and decarboxylated in the presence of aqueous acetic and sulfuric acids according to the method previously employed for the ketonic cleavage of certain beta-keto esters3. The present method appears to be one of the best for the preparation of certain higher aliphatic or aliphatic-aromatic4 methyl ketones [...].

Experimental

Diethyl acylmalonates were prepared by a modification of the procedure of Lund2,5

In a 500-ml three-necked flask equipped with a mercury-sealed stirrer, dropping funnel, and reflux condenser protected by a drying tube, was placed 5.35 g. (0.22 mole) of magnesium. Five ml of absolute ethanol and 0.5 ml of carbon tetrachloride were added. The reaction, which started almost immediately, was allowed to proceed for a few minutes and 75 ml of absolute ether was then added cautiously. The resulting mixture was placed on the steam bath and a solution of 35.2g (0.22 mole) of diethyl malonate, 20 ml of absolute ethanol and 25 ml of absolute ether was added at such a rate that rapid refluxing was maintained, heat being applied when necessary. The mixture was refluxed for three hours, or until the magnesium had dissolved. To the clear solution was added with vigorous stirring6 an etheral solution of 30.8g (0.20 mole) phenylacetyl chloride and the mixture refluxed for one-half hour. The reaction mixture was cooled and acidified with dilute sulfuric acid. The ether phase, with which an ether extract of the aqueous phase was combined, was washed with water and the solvent distilled.

To the crude diethyl phenylacetylmalonate was added a solution of 60 ml of glacial acetic acid, 7.5 ml of concentrated sulfuric acid and 40 ml of water, and the mixture refluxed for four or five hours until the decarboxylation was complete. The reaction mixture was chilled in an ice-bath, made alkaline with 20% sodium hydroxide solution, and extracted with several portions of ether. The combined ethereal extracts were washed with water, dried with sodium sulfate followed by Drierite, and the solvent distilled. The residue containing the ketone was distilled in vacuo to give Phenyl-2-Propanone in 71% yield (bp 97-98.5°C/13 mmHg, 214-215°C/760mmHg).


References
[1] Metzner prepared phenylacetone by this method, but no yield was reported: Ann. Chem. 298, 378 (1897) (https://www.rhodium.ws/pdf/p2p.phenylacetylmalonate.pdf)
[2] Lund, Chem. Ber. 67B, 935 (1934)
[3] Hudson and Hauser, J. Am. Chem. Soc. 63, 3163 (1941)
[4] We believe that the malonic ester method is more convenient for phenylacetone than those described in Org. Synth. Coll. Vol. II, 389, 391 (1943) (http://www.orgsyn.org/orgsyn/prep.asp?prep=cv2p0389)
[5] The present procedure is similar to that of Breslow, Baumgarten, and Hauser [J. Am. Chem. Soc. 66, 1286 (1944)] for the preparation of ethyl tert-butyl acylmalonates.
[6] In certain cases a viscous mixture was formed and unless it was stirred vigorously, lower yields were obtained.

The Hive - Clandestine Chemists Without Borders
 
 
 
 
    Mendeleev
(Stranger)
08-30-04 16:32
No 528275
      Wow this is great, I'm really liking the Pd...     

Wow this is great, I'm really liking the Pd catalyzed alpha arylation and the bromobenzene with propylene oxide to P2Pol and then oxidized to P2P.  Several questions though, is 1-Phenyl-2-nitropropene a regulated precursor?  I didn't find it in the DEA handbook although all of its precursors like benzaldehyde and nitroethane were regulated.  Secondly, I assume the procedure with calcium acetate can be done directly with phenyl acetic acid not its calcium salt, no?  Also does anybody else think some of these are a bit impractical except as a possible laboratory curiosity?  I saw a couple of procedures where the precursors aren't even commercially available, not regulated, it's just that the lab companies simply don't have them.

Trogdor was a man.  A dragon man.  Or maybe just a dragon...
 
 
 
 
    armageddon
08-30-04 18:32
      um..
(Rated as: Check your facts + Leave the moderation to the moderators)
    
 
 
 
    Rhodium
(Chief Bee)
09-01-04 19:49
No 528916
User Picture 
      is 1-Phenyl-2-nitropropene a regulated ...     

is 1-Phenyl-2-nitropropene a regulated precursor?  I didn't find it in the DEA handbook although all of its precursors like benzaldehyde and nitroethane were regulated.

It is not explicitly regulated on a national level as far as I know*, but due to the noxious and unstable nature of nitroalkenes (not to mention a rather low licit demand), they are seldom offered for sale from chem suppliers. But even if the sale of 1-Phenyl-2-nitropropene isn't explicitly prohibited, it sure is a suspicious item and commercial aquisition should be avoided.

Secondly, I assume the procedure with calcium acetate can be done directly with phenyl acetic acid not its calcium salt, no? 

That is not a good idea, as the phenylacetic acid may then react with the calcium acetate to form calcium phenylacetate and free acetic acid, which under the temperatures employed will evaporate and escape.

A lowered ratio between the Ca-Phenylacetate/Ca-Acetate leads to an increased formation of undesired dibenzyl ketone at the expense of less phenylacetone. The loss of acetic acid might be compensated for by the addition of more calcium acetate to the mixture, but I fail to see the advantage of using such a workaround compared to preparinging the proper calcium phenylacetate salt beforehand.

Also does anybody else think some of these are a bit impractical except as a possible laboratory curiosity?

Surely a lot of the procedures are obsolete and byzantine, but this compilation is intended to be comprehensive, rather than strictly practical.


* Some US states regulates 1-Phenyl-2-nitropropene though, see http://www.state.co.us/gov_dir/leg_dir/olls/sl1996/sl.295.htm

The Hive - Clandestine Chemists Without Borders
 
 
 
 
    armageddon
(Hive Bee)
09-01-04 20:54
No 528927
User Picture 
      OK, answering questions is Rhod's privilege.. ;^)     

OK Rhodium I surely would_ve said nothing when you just would've told me to not play the moderator, as I can understand this quite well; but since you tried to convey the impression that I said something wrong, I simply have to reply on that! Here we go...


About "Check your facts":

The "fact" I gave was: "Why do you think a pretty large number of bees use to prepare their own nitrostyrenes!? Better don't try to obtain them through ofiicial sources!"

The "fact" you gave is: "It is not explicitly regulated on a national level as far as I know*, but due to the noxious and unstable nature of nitroalkenes (not to mention a rather low licit demand), they are seldom offered for sale from chem suppliers. But even if the sale of 1-Phenyl-2-nitropropene isn't explicitly prohibited, it sure is a suspicious item and commercial aquisition should be avoided."


Now tell me the difference (besides redundant additional info about the noxious nature of nitropropenes and the low licit demand thing)!

(BTW sorry to interrupt Aurelius' compilation with that;
it doesn't seem to matter anyway)


And about "leave the moderation to the moderators":

WOW! whatta rating! Never seen that before... - but why do you think I would moderate something by stating "I think someone has gone off-topic"?

Greetz A

Pleased to meet you hope you get my name.
But whats puzzlin you is the nature of my game!
 
 
 
 
    Rhodium
(Chief Bee)
09-01-04 23:58
No 528983
User Picture 
      Opposed to me, you answered none of his questions     

Now tell me the difference (besides redundant additional info about the noxious nature of nitropropenes and the low licit demand thing)!

The difference is that you did not answer the question directly - you replied with a question and gave some general advice.  What you wrote might imply that 1-phenyl-2-nitropropene actually would be a regulated precursor, which it isn't in most parts of the world, including the EU and in US Federal legislation - just as I wrote in my post above.

My reply also included an example of an exception to this general rule, as well as offering an explanation as to why 1-phenyl-2-nitropropene seldom is found commercially, even though it is not explicitly prohibited to sell.

(BTW sorry to interrupt Aurelius' compilation with that; it doesn't seem to matter anyway)

If you have any further comments, please PM me - this is not the place for a discussion like this. If you are going to publicly argue about every rating you get in any of the forums, I will have to remove your posting privilegies in the chemistry forums, as you are severely cluttering them with posts which constantly are on the verge of being fully insignificant - your two latest posts in this thread does not really add much substance to the topic at all...

[Note to all: These off-topic posts will be removed in due time, as to not clutter this compilation forever...]

WOW! whatta rating! Never seen that before... - but why do you think I would moderate something by stating "I think someone has gone off-topic"?

I believe it is because you want to boost your ego by telling even fresher newbees than yourself about what you feel is appropriate behaviour around here. However, such advice is solely up to moderators and admins to dole out, as those are the only ones who are in a position to decide about appropriate bee-haviour.

If I catch you arguing about ratings or other severely off topic things in the chemistry forums again, prepare to have your posting privilegies suspended for a time. Ratings are off topic to discuss in all the chemistry forums, and should only be discussed in General/Couch/PM, if at all.

The Hive - Clandestine Chemists Without Borders
 
 
 
 
    Shane_Warne
(Hive Bee)
09-03-04 05:37
No 529281
      A lowered ratio between the ...     

A lowered ratio between the Ca-Phenylacetate/Ca-Acetate leads to an increased formation of undesired dibenzyl ketone at the expense of less phenylacetone.

Rhodium, do you think that an even greater ratio of Ca-Acetate: Ca-phenylacetate than was reported in the page at your site, in experiment 5, would increase the yield of P2P further?

Also, do you think replacing the CO2 stream with vacuum, to remove the ketones as they are formed would slow the reaction, or cause it to require higher temperatures?

The boiling sulfur bath means 444C, right?
 
 
 
 
    Rhodium
(Chief Bee)
09-03-04 18:40
No 529391
User Picture 
      P2P from Ca acetates     

Rhodium, do you think that an even greater ratio of Ca-Acetate: Ca-phenylacetate than was reported in the page at your site, in experiment 5, would increase the yield of P2P further?

I'd say that a further small increase might be possible, but not much. Compare with the table with the PAA:Ac2O ratios at https://www.rhodium.ws/chemistry/phenylacetone.html#phenylacetic - the yield of P2P increases to a certain point, but there is almost no difference in yield when the ratio (w/w) is increased from 1:6 to 1:10 and at 1:16 the yield is actually lowered.

Now the calcium salt pyrolysis is not exactly the same as the one refluxing PAA:Ac2O:NaOAc - but the general trend should still be the same. The highest ratio (w/w) between Ca-Acetate:Ca-phenylacetate used in the article is 1:1 - if you try the reaction with a 2:1 ratio you will see if that favors higher yield. At some point the small yield increase of P2P is not worth the added work (or cost) of a higher salt ratio, so don't overdo it.

Also, do you think replacing the CO2 stream with vacuum, to remove the ketones as they are formed would slow the reaction, or cause it to require higher temperatures?

The inert gas cannot be replaces with a vacuum, as the evaporation rate (or the internal temp) would be screwed up at lower pressures. Feel free to exchange the CO2 for any other inert gas though, such as He, Ar or N2.

The boiling sulfur bath means 444C, right?

Yes, I believe so.

The Hive - Clandestine Chemists Without Borders
 
 
 
 
    Shane_Warne
(Hive Bee)
09-05-04 12:22
No 529678
      Thanks Rhodium. That's unfortunate, the vacuum     

Thanks Rhodium.

That's unfortunate, the vacuum would evacuate it much faster out of the danger zone.

Rhodium this is from Xtaldocs interpretation of the Pb variation:


Preliminary experiments showed that the reaction itself initiated at a lower temp than that which was sustained during the roast. So, the first modification was to run it under a partial vacuum (5" Hg suction on a standard gauge).



So a very small vacuum might be ok? smile

Do you know how the barium reaction temperature is likely to compare with the calcium and Pb?

 
 
 
 
    Rhodium
(Chief Bee)
09-05-04 17:32
No 529704
User Picture 
      vacuum & barium     

So a very small vacuum might be ok?

And a very small vacuum that is...  Xtaldoc seems to refer to a relative pressure drop of only 5" from atmospherical pressure - a mere 633 mmHg (1 atm = 760 mmHg). I'd say that such a vacuum is fully OK to use at it will only facilitate the vapor flow, not change the bp noticeably.

Do you know how the barium reaction temperature is likely to compare with the calcium and Pb?

Seeing that the properties of Barium salts are far more similar to Calcium salts than to Lead(II) salts, I'd wager that the reaction temperature using Barium salts would be inbetween the two, but probably closer to 444°C than to the mere ~200°C required by the Lead(II) method. As Barium is a lot more toxic than calcium, there is no real advantage to the use of such salts.

The Hive - Clandestine Chemists Without Borders
 
 
 
 
    Shane_Warne
(Hive Bee)
09-07-04 11:16
No 530107
      And a very small vacuum that is...     

And a very small vacuum that is...  Xtaldoc seems to refer to a relative pressure drop of only 5" from atmospherical pressure - a mere 633 mmHg (1 atm = 760 mmHg). I'd say that such a vacuum is fully OK to use at it will only facilitate the vapor flow, not change the bp noticeably.

It is small, and of course there's no guarantee it will work the same for the calcium variation.

A combination of inert gas and vacuum might also be a possible option. or just the inert gas like the write-up says.(going around in circles) laugh


Seeing that the properties of Barium salts are far more similar to Calcium salts than to Lead(II) salts, I'd wager that the reaction temperature using Barium salts would be inbetween the two, but probably closer to 444°C than to the mere ~200°C required by the Lead(II) method. As Barium is a lot more toxic than calcium, there is no real advantage to the use of such salts.

That makes sense. Plus, CaCO3  is a lot cheaper.
 
 
 
 
    Rhodium
(Chief Bee)
10-03-04 22:31
No 534323
User Picture 
      Japanese Syntheses of Phenylacetone
(Rated as: excellent)
    

Here follows several japanese syntheses of phenylacetone, as summarized in Chemical Abstracts.

CA Nomenclature note: Analogous to the shorthand Ac for Acetyl (Me[/sub]CO-), Bz is short for Benzoyl (PhCO-) and not Benzyl (PhCH2-)




Reaction of α-halocarbonyl compounds with Grignard reagents. II. Reactions of α-chloroacetophenones.
Teiichi Ando, CA 54. 449215 [Yuki Gosei Kagaku Kyokaishi 17, 777-82 (1959)]

Reactions of some α-chloroacetophenones (RCOCH2Cl) with Grignard reagents in ratio of 1:1 and 1:2 were studied and compared (cf. CA 53, 17971d)

Ratio of 1:1: To a Grignard reagent prepd. from 17.3g PhBr, 2.7 g Mg, and 100 mL Et2O was dropped 15.6g α-chloroacetophenone (I) in 100 mL C6H6 at -3°C to 0°C, the mixt. stirred at 0°C for 30 min, concd. till the bp of the mixt. increased to 72°C, and then refluxed 4 hrs., cooled, decompd. with ice and HCl, and the sepd. org. solvent layer distd. to give 40% deoxybenzoin, bp 160-170°C/6 mmHg, mp 55-56°C (MeOH); semicarbazone mp 147-148°C. Similarly were carried out the following reactions. I with MeMgI, 45% phenylacetone (101-104°C/23mmHg; semicarbazone m. 189-190°C). α-chloro-p-fluoroacetophenone (III) with MeMgI, 41% p-fluorophenylacetone (120-130°C/30mmHg; semicarbazone mp 199-200°C).

Ratio of 1:2: I (7.7g) in 100 mL C6H6 was treated with Grignard reagent prepd. from 17.3g PhBr, 2.7g Mg, and 100 mL Et2O to give 44% 1,2,2-triphenylethanol, mp 87-94°C (ligroin), and 1.8g triphenylethylene, bp10 200-120°C, mp 67-68°C (EtOH). Similarly were carried out the reaction between III with p-F-PhMgBr. 72% tris(p-fluorophenyl)ethylene (bp 195-205°C/12 mmHg, mp 71-72°C).

These results showed that ketones were obtained when the molar ratio of the reactants was 1:1, and 1,1,2-trisubstituted ethanols were obtained when that ratio was 1:2. Based on these experimental results and on the conception of migratory aptitude of atomic groups, the mechanism of the reaction was also discussed.
______ _____ ____ ___ __ _

Reduction of 1-phenyl-1,2-propanedione. I. Synthesis of phenylacetone.
Genshun Sunagawa and Kichihei Okuda, CA 46, 11146a(1952) [J. Pharm. Soc. Japan 72, 117-118 (1952)]

Catalytic reduction of 18.5 g BzAc (I) in 40 mL MeOH with PtO2 gives 11.9 g. PhCH(OH)CH(OH)Me (II) mp 92-93°C; catalytic reduction of a mixt. of I and MeNH2 in MeOH with PtO2 gives dl-ephedrine·HCl, and the mother liquor taken up with ether and distd. gives II.

Refluxing 20 g II with 200 g 20% H2SO4 5-7 h, taking up with ether, and distg. give 13g PhCH2Ac; semicarbazone, mp 190-6°C.
______ _____ ____ ___ __ _

Phenylethynylcarbinol and its derivatives. II. Synthesis of phenylacetone
Ichiro Hirao, Chem. Abs. 10534c (1954) [J. Chem Soc. Japan, Ind. Chem. Sect. 56, 265-266(1953)]

Commercial PhCH(OH)CCH was hydrated with H2SO4 to PhCH(OH)COCH3, which was hydrogenated with Raney-Ni catalyst by H2 of 50 atms. in an autoclave at room temp. The white crystals obtained correspond to 1,2-dihydroxy-1-phenylpropane (I) of mp 91-92°C; yield, 95% PhCH2Ac was prepd. by heating of I with H2SO4, oxalic acid, H3PO4, etc.; yield approx. 73%.
______ _____ ____ ___ __ _

Synthesis of phenylacetone
Keiichi Shishido and Shigeru Kukita
CA 42, 6339g [J. Soc. Chem. Ind. Japan 48, 34(1945)]

A Grignard reagent prepd. from 6g Mg, 1.70 mL anhyd. ether, a small crystal of I2 and 32 g PhCH2Cl was dropped slowly into 20g AcCl in 150 mL ether, kept at -10°C to -15°C, and the product treated with ice-cooled concd. HCl, extd. with ether, neutralized, and distd. under 9 mmHg at 87-93°C, giving 27% PhCH2Ac, mp 188.5-189°C from alcohol).

This one is a little fishy - the mp data makes no sense, as P2P is a liquid. It does however jive with the mp of P2P semicarbazone, which is mentioned in two of the above abstracts as well. I wonder if it's a typo or a mistaken product?

The Hive - Clandestine Chemists Without Borders
 
 
 
 
    bio
(Hive Bee)
10-04-04 22:59
No 534445
      Two Products One Precursor?     

Am I reading this correctly that the Dione gives both ephedrine and P2P (via the Diol)? What would be the easiest way to make the Diol (1 Phenyl 1,2 Propanediol)?


.........Reduction of 1-phenyl-1,2-propanedione. I. Synthesis of phenylacetone.
Genshun Sunagawa and Kichihei Okuda, CA 46, 11146a(1952) [J. Pharm. Soc. Japan 72, 117-118 (1952)]

Catalytic reduction of 18.5 g BzAc (I) in 40 mL MeOH with PtO2 gives 11.9 g. PhCH(OH)CH(OH)Me (II) mp 92-93°C; catalytic reduction of a mixt. of I and MeNH2 in MeOH with PtO2 gives dl-ephedrine·HCl, and************************the mother liquor taken up with ether and distd. gives II.*****

Refluxing 20 g II with 200 g 20% H2SO4 5-7 h, taking up with ether, and distg. give 13g PhCH2Ac; semicarbazone, mp 190-6°C.
 
 
 
 
    psychokitty
(«»)
10-05-04 02:02
No 534475
      Two more P2P syntheses     

Take from the following thread:

Post 108587 (dormouse: "Two new MD-P2P synths for y'all to chew on!  -psychokitty", Novel Discourse)


Author  Topic:   Two new MD-P2P synths for y'all to chew on!
psychokitty
Member   posted 12-14-98 09:27 PM         
--------------------------------------------------------------------------------
The following information was taken from these three references:
J.C.S. Perkin I, 1974 pp1727-1731.
J.C.S. Perkin I, 1975 pp1548-1551.
Chemical Communications, 1971, pp818-819.

These experiments originally used methyleugenol to synthesize 3,4-DM-P2P, but so what? They should work equally well on safrole to make MD-P2P.

(a). Hydroxlation of methyeugenol to DM-P2Pol.

Methyleugenol (133mg) dissolved in tetrahydrofuran (3ml) was added to a solution of mercury(II)acetate (400mg) in tetrahydrofuran-water (1:2,4.5ml) and the mixture was stirred at room temperature for 24hr. After addition of 3M-sodium hydroxide (4ml) followed by 0.5M-sodium borohydride in 3M-sodium hydroxide (4ml), mercury was allowed to settle and filtered off; solid sodium chloride was then added to saturate the water layer. The upper layer and ethereal extracts (3 times 10ml) of the lower one were pooled, dried (Na2SO4), and evaporated. The residue (123mg), homogenous by GLC, was crystallized from ether-light petroleum (bp40-60degC)and shown to be DM-P2Pol.

(b) DM-P2Pol oxidation

DM-P2Pol was dissolved in acetone (1ml) and oxidized with an excess of Jones reagent at 0degC for 5min. After the addition of isopropyl alcohol (0.5ml) followed by saturated aqueous sodium hydrogen carbonate (2ml), the mixture was extracted with chloroform (5 times 10ml). The residue after evap. of solv. was DM-P2P (25mg).

The above synthesis is pretty cool, but in my opinion, inferior to the Wacker analogue reaction using mercuric acetate and Jones reagent that used to be #2 in TSI.

Now for the real goods:

Sythesis of DM-P2P directly from methyleugenol using PdCl2-LiCl/Hg(OAc)2/CuCl2.

Mercury(II)acetate (8.96g)and methyleugenol (5g) were stirred at 25degC for 20min in MeOH (180ml)and then added to a stirred solution of lithium chloride (238mg), palladium chloride (493mg), and copper(II)chloride (14.4g)in MeOH. The mixture was heated under reflux for 1hr, then poured into saturated sodium hydrogen carbonate solution (500ml); the solution was filtered and heated under reduced pressure to remove MeOH. The residual solution was extracted with ether (6 tms 100ml) and the extract was dried and evaporated. Distillation of the oil so obtained (5.2g) under vacuum afforded DM-P2P (3.6g), bp 120-123 at 0.6 mmHg.

Hey, Strike! I wanna be in the book!

The last synthesis above is pretty cool, no? MD-P2P in one hour from safrole. Only thing that suck is the amount of solvent needed for scaling up. Maybe it can be reduced. Anyone have any comments out there in da Hive?

Before I forget to mention, what is actually happening here is that the MeOH-HgOAc add to the double bond of the alkene and then the PdCl2 swaps places with the HgOAc, which forms an unstable intermediate that rearranges to the final ketone. I wonder what would happen if you used PdCl2 in the aminomercuration synth. Would the PdCl2 swap with the HgNO3, forming an unstable intermediate, to then eventually form -- what? MDMA?

Oh, well. It's just an idea.
 
 
 
 
    psychokitty
(«»)
10-06-04 02:44
No 534615
      Synthesis of p-fluorobenzyl methyl ketone
(Rated as: good read)
    

As originally reported by Rhodium in the following thread:

Post 286777 (Rhodium: "p-Fluorophenyl-2-propanone", Novel Discourse)


and, in more general terms, by Regis in the following thread:

Post 365747 (Regis: "Arylmagnesium bromides/chloroketones reaction", Novel Discourse)


Some Fluorinated Amines of the Pressor Type
C. M. SUTER AND ARTHUR W. WESTON
JACS Vol. 63, pp. 602-605 (1941)



Experimental example of the Grignard synthesis of p-fluorobenzyl methyl ketone:

p-Fluorobenzyl Methyl Ketone.-A Grignard reagent was prepared from 35 g. (0.20 mole) of p-fluorobromobenzene and 4.6 g. (0.19 mole) of magnesium. To this was added 18.5 g. (0.2 mole) of chloroacetone in 50 ml. of ether as rapidly as the refluxing of the reaction mixture would allow. The ether was removed by heating the reaction flask in an oil-bath and at 100 'C the residue foamed with formation of a gel from which ether was removed slowly by heating at 135-140'C for forty-five minutes. The flask was cooled, ice and dilute acid were added, the heavy oil which separated was removed with ether and the ether solution dried and fractionated. There was obtained 11.2 g. (37%) of practically pure ketone distilling at 106-107'C (18 mm.).

A variation of this procedure in which the reaction mixture was not heated above 100'C gave a product that could not be satisfactorily fractionated. Substantially pure ketone was finally obtained by conversion to the sodium bisulfite addition compound followed by regeneration with dilute sodium carbonate. It distilled at 108'C (18 mm.) as a light yellow oil with +'D 1.4965, d*O, 1.107, MD calcd. 40.07, obsd. 40.17. After a few days of standing, crystals of p-fluorobenzoic acid were deposited. Carbon and hydrogen analyses gave low results even with material kept in a sealed ampoule. A solid derivative was therefore prepared. The dinitrophenylhydrazone did not form but the semicarbazone, m. p. 200.5-201.5', separated readily. Analysis by the Jamieson method of titrating with potassium iodate gave only fair results. Anal. Calcd. for C10H12ON3F: eq. wt., 52.30. Found: eq. wt. 51.50. In this analysis the end-point was not stable, probably due to the reaction of the liberated ketone with iodine.
 
 
 
 
    lugh
(Moderator)
10-16-04 00:52
No 536024
      Yet Another Old P2P Synthesis
(Rated as: good read)
    

Another old p2p synthesis, a Claisen condensation of phenylacetyl chloride using zinc-methyl iodide in toluene-ethyl acetate yielding 72%; Ber 58 339-41 (1925) wink



cool

Chemistry is our Covalent Bond
 
 
 
 
    Organikum
(Wonderful Personality)
10-16-04 08:10
No 536062
      Translation     

Translation:

Preparation of methylbenzylketone:
54g phenylacetylchloride dissolved in the same volume of toluene was added to a solution of methyl-zink-iodide, which was prepared from 32,5g zinc (as zinc/copper couple) and 71g methyliodide in toluene and 16ccm ethylacetate ("Essigester"). Yield: 34g, 72% of theory.

Earlier in the article the preparation of phenylacetylchloride is mentioned:
...was prepared in quantitative yields from phenylacetic acid and thionylchloride....

P2P is only a precursor in the experiments of the authors and so they keep it short.

Earlier it is mentioned that hydrolysis of 3-phenyl-propionylacetone, C2H5.CO.CH(C6H5).CO.CH3 with 20% aqueous NaOH solution yields a mixture of ethyl- and methyl- benzylketone

so near, so far......
 
 

All 91 posts   End of thread   Top
   

 https://the-hive.archive.erowid.org    the-hive@erowid.org
   
Powdered by Black Tea(R) Version 5.02.5 Platinum Ed., (c) 2018 - 2023, LWR Labs Corporation Asia. All rights reserved.

Links     Erowid     Rhodium

PIHKAL     TIHKAL     Total Synthesis II

Date: 05-03-24, Release: 1.6 (10-04-15), Links: static, unique